· web viewoncerned ‘tax bonus payments’ made by rudd government. argued that...

60
UQ Semester 1, 2014 LAWS1116 Constitutional Law Week 1: Introduction and Separation of Powers Reasons/Benefits of SOP o Serves rule of law by making government subject to law. o Provides for checks and balances on the power of government bodies. Perfect SOP impractical and impossible to achieve. Judicial independence much stronger than separation between other branches. SOP much more relaxed at State level. Parliaments have much greater influence on State courts; however HC has placed some limits (ie Kable). Dignan’s Case (1931) o High Court has allowed extensive delegation of legislative power to the executive with few meaningful limits. o Section upheld because: (1) Wide delegation said to be consistent with British constitutional practice (and we adopted their model). (2) If not allowed, effective government would be impossible (especially considering nature of modern state). Parliament has neither the time nor expertise. (3) Doctrine of responsible government ensures final oversight remains with legislature. o Court recognized some minimal limits: Delegation cannot be so wide that it is not a law with respect to any head of power (Dixon J). No special limit here, just saying that delegation is subject to the same limits that legislative power generally is subject to. Cannot have a general abdication of legislative power on a particular head of power (Evatt J). Huddart Parker v Moorehead (1909) o Griffith CJ’s definition of judicial power- leading authority. o (1) Controversy or dispute ss 75-78- power to decide “matters” Limits law-making power of judiciary- can only decide cases that come before them. Re Judiciary Act (1921)- providing hypothetical opinions (in absence of dispute) is not judicial power. Also, Chapter III exhaustively sets out judicial power of Commonwealth. Courts cannot exercise other judicial power. 1

Upload: vodung

Post on 13-Mar-2018

213 views

Category:

Documents


0 download

TRANSCRIPT

Page 1: · Web viewoncerned ‘tax bonus payments’ made by Rudd government. Argued that payments were unsupported by a head of power. Upheld legislation as a valid exercise of s 61, together

UQ Semester 1, 2014

LAWS1116 Constitutional LawWeek 1: Introduction and Separation of Powers

Reasons/Benefits of SOPo Serves rule of law by making government subject to law.o Provides for checks and balances on the power of government bodies.

Perfect SOP impractical and impossible to achieve. Judicial independence much stronger than separation between other branches.

SOP much more relaxed at State level. Parliaments have much greater influence on State courts; however HC has placed some limits (ie Kable).

Dignan’s Case (1931)o High Court has allowed extensive delegation of legislative power to the executive

with few meaningful limits. o Section upheld because:

(1) Wide delegation said to be consistent with British constitutional practice (and we adopted their model).

(2) If not allowed, effective government would be impossible (especially considering nature of modern state). Parliament has neither the time nor expertise.

(3) Doctrine of responsible government ensures final oversight remains with legislature.

o Court recognized some minimal limits: Delegation cannot be so wide that it is not a law with respect to any head of

power (Dixon J). No special limit here, just saying that delegation is subject to the same limits that legislative power generally is subject to.

Cannot have a general abdication of legislative power on a particular head of power (Evatt J).

Huddart Parker v Moorehead (1909)o Griffith CJ’s definition of judicial power- leading authority.o (1) Controversy or dispute

ss 75-78- power to decide “matters” Limits law-making power of judiciary- can only decide cases that come before

them. Re Judiciary Act (1921)- providing hypothetical opinions (in absence of

dispute) is not judicial power. Also, Chapter III exhaustively sets out judicial power of Commonwealth.

Courts cannot exercise other judicial power. Hilton v Wells (1985)- issuing phone-tapping warrants is not judicial power.

See Grollo v Palmer (1995) for persona designata.o (2) Impact on [existing] rights relating to life, liberty or property.

Existing rights: Tasmanian Breweries Case (1970). Facts as determined applied to law as determined.

Vagueness of standards can mean that power is non-judicial: Albarran’s Case (2007).

Basic rights: R v Quinn (1977) per Jacobs J.1

Page 2: · Web viewoncerned ‘tax bonus payments’ made by Rudd government. Argued that payments were unsupported by a head of power. Upheld legislation as a valid exercise of s 61, together

UQ Semester 1, 2014 Decision appears to be one of convenience. Clearly power to deregister trademarks is just a historical exception. Widely regarded as a mistaken decision, and basic rights have not

been invoked since then.o (3) Decision must be conclusive.

Binding, authoritative decision. Does not mean that it cannot be appealed. Means that in the absence of a

direct appeal, the decision is immediately binding on parties. Not conclusive if it can be undermined in any of following ways:

(1) Collateral attack- indirectly challenged in different proceedings at same-level court.

(2) De novo rehearing- decision is subject to complete rehearing (on facts and law) before any practical effect.

o Brandy’s Case (1995)- number of features of de novo rehearing (1) Earlier decision not enforceable without complete

rehearing (not automatically enforceable). (2) Rehearing starts from beginning- all witnesses and

submissions reconsidered. (3) Reviewing body hears matter entirely, on both facts

and law. In Brandy: decision was not subject to automatic

rehearing (it was merely an option), so still judicial power.o Upshot- if decision subject to de novo rehearing, it can be

exercised by a non-judicial body. To be conclusive must abide by the following doctrines:

(1) Res judicata (2) Functus officio

o Textbook: (4) Non-consensual Cannot opt out of a judicial decision. Court can bind you to its judgment,

whether or not you consent. Arbitration looks like judicial power, but is usually based on a contract that

was consensually entered into. Historical factors

o R v Davison (1954)- issuing of voluntary sequestration order in bankruptcy law is judicial power even though no dispute.

o Cominos v Cominos (1972)- power to make maintenance orders following marriage breakdown is judicial, even though large policy discretion.

o Thomas v Mowbray (2007)- Power to issue ‘interim control orders’ is judicial due to serious consequences, despite broad discretion.

o Quinn may fit here?o Best regarded as authority that historical factors can be considered when

determining whether a power is judicial. Thornton, R v Davison- executive/legislative functions can be exercised by courts if they

are incidental to the exercise of judicial power.

2

Page 3: · Web viewoncerned ‘tax bonus payments’ made by Rudd government. Argued that payments were unsupported by a head of power. Upheld legislation as a valid exercise of s 61, together

UQ Semester 1, 2014

Week 2: Separation of Powers II HC has detailed rules to separate judicial power from non-judicial- rules govern both who

can exercise it and how. Prohibitory and permissive rules. Federal judicial power cannot be vested in a body not designated in s 71 (HCA, federal

courts, SSCs) Interpretation dates back to Wheat Case (1915) and reiterated in Boilermakers Case

(1956). Confirmed by PC in A-G (Cth) v Queen (1957). Alexander’s Case (1918)

o Federal judicial power can only be vested in a court in the strict sense of the term- primary function is judicial.

o Has been superseded in regards to federal courts, but still applies to State courts exercising federal jurisdiction.

Boilermakers Case (1956)o Non-judicial powers cannot be vested in federal courts.o Federal courts must be exclusively judicial in function.o Thornton, R v Davison- executive/legislative functions can be exercised by courts if

they are incidental to the exercise of judicial power. Chapter III courts must be correctly constituted under s 72- requirements designed to

safeguard judicial independence. Illustrated in Alexander’s Case. Barton and Powers JJ held a judge already holding tenure

cannot be appointed to another court unless he/she also enjoys tenure on the second body.

Delegation Judicial Powero Court may not abdicate federal judicial power, but may delegate under certain

conditions.o Harris v Caladine (1991)- court may delegate judicial power to non-judicial officer.

Majority required two conditions: (1) Delegation must not be so extensive that the judges no longer constitute

the court. (2) The decisions of the delegates must be subject to review or appeal by the

judges on both facts and law. [However then it is no longer conclusive so no longer judicial power].

McHugh and Dawson JJ required a de novo rehearing. Composition of State courts

o Kotsis v Kotsis (1970)- state courts exercising federal judicial power cannot delegate power.

3

Page 4: · Web viewoncerned ‘tax bonus payments’ made by Rudd government. Argued that payments were unsupported by a head of power. Upheld legislation as a valid exercise of s 61, together

UQ Semester 1, 2014o Confirmed in Knight v Knight (1971).o Overturned in HCF Case (1982):

Federal parliament must take state courts as it finds them. No reason why a non-judicial officer may not form part of a state court.

State courts may include acting judges with limited terms, but independence and impartiality must be maintained: Forge v ASIC (2006).

Bills of attainder: Parliament cannot require the courts to enforce bills of attainder or dictate the way courts exercise judicial power.

o Polyukhovich v Commonwealth (1991)- no blanket ban on retrospective legislation, but a law may be invalid if it is clearly aimed at a specific person (thus is a legislative judgment passed after the fact).

Kable principleo State courts vested with federal judicial power may not hold functions incompatible

with that power. Where public confidence would be undermined.o Revisited in Fardon v A-G (Qld) (2004)- upheld legislation. Distinction from Kable

was that Act did not apply to named person, but allowed Supreme Court to make an independent assessment.

o Revived in series of HC decisions.o SA v Totani (2010)- legislation left no discretion to judge.o Wainohu v NSW (2011)- Judge not bound by rules of evidence and need not give

reasons (limits scope for judicial review). Grollo v Palmer conditions not met. Judicial Power outside Ch III

o Such judicial power cannot be vested in federal courts.: Re Wakim; Ex parte McNally (1999).

Cross-vesting legislation giving federal courts state judicial power.o Re Judiciary Act illustrates same principle but that was clearly non-judicial power.

Inherent jurisdictiono Parliament cannot remove jurisdiction conferred upon courts by the Constitution.o Right to appeal to HC with leave (s 73) cannot be abolished: Cockle v Isaksen

(1957).o Original jurisdiction under s 75 cannot be removed: Lim v Minister for Immigration

(1992). Privative clauses

o May limit scope of judicial review, but may not oust jurisdiction.o Plaintiff S157/2002 v Cth (2003)- such clauses will be construed narrowly and

cannot remove ability of courts to review decisions for jurisdictional error.o Similar rule applies in States: Kirk v Industrial Court of NSW (2010).

Persona designata ruleo Non-judicial power can be vested in judge in his/her personal capacity. First found in

Hilton v Wells.o In Grollo v Palmer HC imposed two conditions:

(1) Consent of judge. (2) Compatibility: function must be compatible with judge’s capacity to

perform his/her judicial functions, taking into account time required and nature of role. If role would undermine public confidence in judge’s independence, not allowed.

4

Page 5: · Web viewoncerned ‘tax bonus payments’ made by Rudd government. Argued that payments were unsupported by a head of power. Upheld legislation as a valid exercise of s 61, together

UQ Semester 1, 2014 Courts martial

o Judicial power with respect to military offences may be exercised by military tribunals not conforming with Ch III.

o Re Tracey (1989)- Can only try military offences. Must show demonstrable connection with military functions.

o Three views: (1) Outside judicial power of Commonwealth. (2) Non-judicial power. (3) Exception.

o First prevailed until Lane v Morrison (2009) where second was endorsed. Court administration

o Courts make rules of procedure. Historically key aspect of judicial rule. Incidental to exercise of judicial power: R v Davison.

Other exceptionso Statutory authorities controlling public service discipline: R v White (1963).o Parliament with respect to its own privileges: R v Richards (1955).

Week 3: Commonwealth-State Relations Three stages of High Court’s approach to intergovernmental immunities

o Dual sovereignty (1903-1920)- extensive immunity of instrumentalities was recognized

o Legislative supremacy (1920-1947)- no immunities enforced.o Dual federalism (1947-present)- immunities revived in a modified form. Balance

struck. D’Emden v Pedder (1904)

o Start of dual sovereignty era. Held states could not burden federal government’s payment of salaries by taxing the receipts- very pedantic decision.

o Constitution made Commonwealth and States sovereign entities. Superiority belongs to Cth- s 109.

o Followed in Deakin v Webb (1904) and Commonwealth v NSW (1906).o In Webb v Outrim (1907), PC rejected D’Emden. In Baxter v Commissioner of

Taxation HC approved D’Emden, criticizing Outrim. Emphasized federal compact and purposive interpretation.

Railway Servants’ Caseo State immunity from Commonwealth laws.o Followed D’Emden to find NSW instrumentalities were not bound.

5

Page 6: · Web viewoncerned ‘tax bonus payments’ made by Rudd government. Argued that payments were unsupported by a head of power. Upheld legislation as a valid exercise of s 61, together

UQ Semester 1, 2014 Exceptions

o Where a Commonwealth power, by its very terms, involves control of some aspect of State government, interference will be permitted.

o Local governments do not enjoy immunity from Commonwealth laws: Municipalities Case (1919).

Engineers’ Case (1920)o Beginning of legislative supremacy era.o Raised similar issues as Railway Servants.o D’Emden was reinterpreted [Commonwealth supremacy] and Railway Servants was

overruled.o Isaacs J’s majority judgment: s 51 must be interpreted using two guiding principles:

(1) the undivided sovereignty of the Imperial Parliament [that enacted CACA] (2) the doctrine of responsible government

o These principles dictate a broad and expansive interpretation of federal powers. It is up to responsible government to keep power in check.

Melbourne Corporation Case (1947)o Commenced HC’s current approach of dual federalism.o Two questions to be asked:

(1) Is the legislation within power? (2) Does the legislation violate any prohibitions? [express or implied]

o Dixon J- leading judgment- Commonwealth has broad powers and can make general laws affecting the

States, but a State immunity against Commonwealth control is implied from the federal system.

Efficacy of the system demands that the Commonwealth may not enact a law aimed at the restriction or control of a State in the exercise of its executive authority.

o Starke J Similar approach- limited State immunity may be implied from the federal

system. Government is a dual system; depends on maintenance of States and their

powers. Legislation that aims to destroy State government or curtails or interferes in

a substantial manner with its exercise of constitutional power will therefore be constitutionally invalid.

o About guaranteeing continued existence and function of States, not so much about sovereignty.

Commonwealth v Cigamatico Clarified application of Melbourne Corporation to Commonwealth immunity.o Majority followed Dixon J in Uther v Federal Commissioner of Taxation (1947), and

held that if any supremacy exists, it lies with the Commonwealth.o States have no power to regulate the rights, privileges, duties or disabilities of the

Cth. Payroll Tax Case

o Commonwealth tax on employers’ wages, including States as employers. Upheld.

6

Page 7: · Web viewoncerned ‘tax bonus payments’ made by Rudd government. Argued that payments were unsupported by a head of power. Upheld legislation as a valid exercise of s 61, together

UQ Semester 1, 2014o Significant that the legislation did not single out the States, but applied to all

employers without discrimination. Therefore passed Melbourne Corporation Test. QEC v Commonwealth (1985)

o Legislation to bring Qld dispute under Cth jurisdiction. Invalid. Specifically directed at Queensland situation. Two limbs of test:

(1) Does the Commonwealth law single out or discriminate against the States? (a particular State or States altogether)

(2) Does the law inhibit the capacity of a State to function as a government? Native Title Act Case

o Act not intended to discriminate. Disproportionate effect due to history/geography. Act does not deprive WA of its ability to function as a government. Second limb protects only following areas:

‘Existence and nature’ of State government, but not its specific powers. ‘Machinery of government’ and the capacity of three branches to exercise

powers. Essential ‘personnel, property, goods and services’ State requires to operate.

Re AEUo No blanket immunity against Commonwealth industrial relations laws but protected

areas: ‘Number and identity of persons’ and ‘term of appointment’ ‘Qualifications and eligibility for appointment’ ‘Number, identity, terms and conditions of employment for higher levels of

government.o Regulating decisions to promote/transfer employees specifically left open.

Industrial Relations Act Caseo Restricted grounds of dismissal. Held to interfere with ‘number and identity’ of

redundancies and conditions of employment provided at the ‘higher levels of government’.

Re Residential Tenancies Tribunal of NSWo States could not make laws that interfered in relations between Commonwealth and

its subjects.o Where Crown (Cth) enjoys position of privilege/immunity State cannot

diminish/regulate the exercise of that privilege/immunity (as in Cigamatic). Where the Crown is equal at law with the citizens, State law cannot single out Cth for differential treatment.

Austin v Commonwealth (2003)o Levy invalid for interfering with ability of States to discharge functions (under AEU).

New test:o Does the Commonwealth law interfere with the capacity of a State to function as a

government?o Possible that mere discrimination may not be enough. Melbourne Corporation and

QEC good law. Clarke v Federal Commissioner of Taxation

o French CJ reformulated test using multifactorial approach using earlier factors:o (1) Does the law single out the states or impose a special burden on them?

[Melbourne Corp]7

Page 8: · Web viewoncerned ‘tax bonus payments’ made by Rudd government. Argued that payments were unsupported by a head of power. Upheld legislation as a valid exercise of s 61, together

UQ Semester 1, 2014o (2) Does it impact on the ability of the states to exercise their powers and

functions? [Payroll tax]o (3) Is the impact of the law clearly authorized by a Commonwealth head of power?

Week 4: Resolution of Conflicts of Laws Characterization

o (1) Examine the constitutional head of power that support the law [may be more than one].

o (2) Examine the correspondence between the law and the power [is it really with respect to it?].

Early HC interpreted powers narrowly to protect State reserve powers dominant characterization.

R v Barger (1908)o Struck down for being outside taxation power and for discrimination between states

(s 51(ii)).o Motives of legislature irrelevant but HC still has to determine true subject matter.o Dominant character was working conditions, not taxation.

Huddart Parker v Mooreheado Corporations power had to be interpreted narrowly to protect States’ reserve

powers.o Dominant character is regulation of trade and commerce.

Engineers’ Caseo Rejection of implied limitations on Cth powers heralded new approach to

characterization.o HC no longer asks about dominant character, but asks whether there is a

reasonable view of the law that places it within the head of power: Multiple characterization.

o If multiple characteristics, only one has to fit under the head of power. Doesn’t have to be primary or main purpose.

o No objection that law touches topic outside Commonwealth head of power: Murphyores v Cth.

Fairfax v Federal Commissioner of Taxation (1965)o Rejected argument that law was not about taxation. Federal powers may extend to

laws that also have another purpose.o Barger was in doubt following Engineers (minority judgments preferred).

NSW v Commonwealth (WorkChoices)o Upheld legislation, declined to use implications to limit corporations power.

8

Page 9: · Web viewoncerned ‘tax bonus payments’ made by Rudd government. Argued that payments were unsupported by a head of power. Upheld legislation as a valid exercise of s 61, together

UQ Semester 1, 2014o Commonwealth heads of power must be read with all the generality that the words

admit.o Kirby and Callinan JJ dissented- power limited by federal nature and other parts of s

51. Conflicts of laws

o Conflict of duties Commonwealth law and a state law will be clearly inconsistent if one of them

requires an act and the other prohibits it. R v Brisbane Licensing Court; Ex parte Daniell (1920): referendum along with

Senate elections.o Conflicts of rights

One law authorizes an act and another law prohibits it. Could obey both. Care must be taken in whether the act is really authorized or just not ruled

out. Colvin v Bradley Brothers (1943)- Cth law permitting women to work on mills

prevailed. Cases have held that a Cth law granting a license to operate a business

overrides state law requiring the grant of a separate state license: O’Sullivan v Noarlunga Meat (1954).

Sometimes have to fulfill 2 requirements: Commercial Radio Coffs Harbour v Fuller (1986):

Applying to different acts/regulating different areas. Upheld.o Overlapping requirements

One law imposes more onerous requirements than the other. Depends on interpretation of less onerous law: is it just designed to set a

minimum standard or is it meant to exhaust the requirements? Clyde Engineering v Cowburn (1926): Commonwealth award was not

intended as a minimum, it was meant to standardize employment arrangements nationally.

o Different Penalties Inconsistency. Commonwealth law prevails. Hume v Palmer (1926): different procedures and penalties for same act.

o Operational Inconsistency Terms do not directly conflict, but exercise of powers may clash. Western Australia v Cth (Mining Act Case) (1999)- no direct conflict, but when

a defence practice area is declared in a mining area an operational conflict will arise. Cth prevails.

Momcilovic v The Queen (2011)- different maximum penalties for drug trafficking.

Treated as a clash of powers. No conflict if state courts exercise their powers in a way consistent with Cth law.

However if State courts were to impose penalties in excess of the federal maximum, a conflict would arise.

o Covering the field If a Cth law shows an intention to cover the field- to comprehensively

regulate a particular topic- then any state law on that topic is overridden.

9

Page 10: · Web viewoncerned ‘tax bonus payments’ made by Rudd government. Argued that payments were unsupported by a head of power. Upheld legislation as a valid exercise of s 61, together

UQ Semester 1, 2014 Sometimes federal law may explicitly state intention. In other cases, scope of

law will provide evidence of intention. See Clyde Engineering and Hume v Palmer.

Viskauskas v Niland (1983)- subject matter of Cth Act suggests that it is intended to be exhaustive and exclusive.

Amendment passed- retrospective declaration that it was not intended to cover the field.

University of Woollongong v Metwally (1984)- this made no difference to outcome. Substance of law matters, not form or declared intentions. Commonwealth cannot circumvent s 109 in this way.

But, a prospective declaration of intent may sometimes have relevance in interpreting the legislation: R v Credit Tribunal (1977).

Recent example: Commonwealth v ACT (2013): Marriage Act 1961 (Cth) intended to cover the field of marriage.

o Consequences of conflict State law is inoperative, and will recommence if inconsistency eliminated. State law inoperative and ineffective from moment inconsistency arises- this

cannot be retrospectively remedied. High Court likely to sever inoperative provisions where possible. Depends on

how integral they are.

Week 5: External Affairs Three aspects of power:

o Geographical power: matters, things or events physically external to Australia.o Treaty power: implementing Australia’s treaty obligations.o International relations power: aimed at preserving international law and friendly

relationships. Geographical Power

10

Page 11: · Web viewoncerned ‘tax bonus payments’ made by Rudd government. Argued that payments were unsupported by a head of power. Upheld legislation as a valid exercise of s 61, together

UQ Semester 1, 2014o Ongoing debate about whether matters need to have demonstrable connection to

Australia.o NSW v Commonwealth (Seas and Submerged Lands Case) (1975)- territorial sea

defined by international law. Extends from low-water mark to 12 nautical miles offshore (22km).

Do States or Commonwealth have jurisdiction? Commonwealth law upheld, on two grounds: (1) law implemented

international conventions (treaty power). (2) law fell within geographical power. Territorial sea is within Australian jurisdiction, but outside Australian landmass (bounded by low-water mark).

o Polyukhovich v Commonwealth (1991)- majority upheld legislation under geographical power. 5 judges held that the exercise of the geographical power did not require connection to Australia.

Treaty Powero Commonwealth v Tasmania (Tasmanian Dam Case) (1983)- Mason, Murphy,

Brennan and Deane JJ upheld legislation. Treaty power not limited by content. Can be used to regulate matters internal to Australia. Existence of treaty shows issue is of ‘international concern’.

o Precursors to Tasmanian Dam R v Burgess; Ex parte Henry (1936): HC upheld Air Navigation Act 1920 (Cth)

under treaty power. Majority thought the power extends to all bona fide international treaties.

Koowarta v Bjelke-Petersen (1982): upheld racial discrimination legislation as it implemented a treaty.

Majority was Mason, Murphy, Brennan and Stephen JJ. Mason, Murphy and Brennan JJ took an expansive view of the power. Any law implementing a treaty may be within federal power.

Stephen J took more moderate view- topic must be of ‘international concern’. Herein lies the reason why Tasmanian Dam is so important- in this case there were 3 on each side with Stephen J in the middle. Tasmanian Dam was an unqualified majority.

o Limits on treaty power (1) Bona fide treaty

Commonwealth cannot rely on treaties entered into merely to confer legislative power- Brennan J in Tasmanian Dam. Also Burgess.

However there is a presumption that all treaties entered into are bona fide.

(2) Nexus requirement Commonwealth law must be reasonably related to the treaty. In

Airlines of NSW v NSW (No 2) (1965)- HC must strictly scrutinize whether law is appropriate and adapted to the implementation of the treaty.

Richardson v Forestry Commission (1988)- whether the legislature could reasonably form the view that the law was necessary to implement the treaty.

o Also unanimously reiterated wide view from Tasmanian Dam.

11

Page 12: · Web viewoncerned ‘tax bonus payments’ made by Rudd government. Argued that payments were unsupported by a head of power. Upheld legislation as a valid exercise of s 61, together

UQ Semester 1, 2014 Victoria v Commonwealth (Industrial Relations Act Case) (1996)-

adopts a middle position- legislation must have a close enough connection to the treaty to be reasonably regarded as an implementation of its provisions.

o “It is for the legislature to choose the means [of implementing the treaty] provided that the means chosen are reasonably capable of being considered appropriate and adapted to that end.” – at provision level not law level

(3) Constitutional limits Power cannot be used to override constitutional rights and guarantees. But HC has generally declined to read down power by reference to

jurisdiction of states. (4) Binding obligations

Binding obligation is required, but the HC will take a flexible approach in determining whether such an obligation exists.

In Tasmanian Dam Brennan J held binding obligation required, but majority disagreed.

Queensland v Commonwealth (Daintree Rainforest Case) (1989)- majority held an obligation is needed. However HC will not strictly scrutinize the treaty to see whether it confers obligations. Defer to views of international community.

Richardson- no current obligation, however HC upheld legislation as it anticipated a possible future obligation [reasonable likelihood of this happening].

Industrial Relations Act Case- must be a reasonably specific obligation. Mere aspiration not enough. Not applied strictly as distinction can be difficult.

International Relations Powero Covers matters falling within doctrine of comity- maintaining friendly relationships

by recognizing other country’s sovereignty and that they can make sovereign acts.o R v Sharkey (1949)- HC upheld prohibition on sedition against British Crown under

doctrine of comity, interest in maintaining friendly relations. Was a wartime decision.

o Status of foreign laws Covers legislation recognizing foreign laws. Also extends to

extradition/marriage etc. Kirmani v Captain Cook Cruises (No 1) (1985)- power extends to legislation

excluding foreign laws that purport to apply in Australia- negative dimension of power.

o Matters of international concern HC has suggested that power may authorize laws on matters of international

concern- Mason, Murphy and Deane JJ in Tasmanian Dam, but not Brennan J. What is of international concern? Mason J issue should be left to

Parliament. Only need an expression of international concern. Brennan J in Polyukhovich- standards in question must be clearly expressed

by international community and broadly adhered to in international practice. Recently backed away- Industrial Relations Act Case- aspiration not enough.

12

Page 13: · Web viewoncerned ‘tax bonus payments’ made by Rudd government. Argued that payments were unsupported by a head of power. Upheld legislation as a valid exercise of s 61, together

UQ Semester 1, 2014 But Thomas v Mowbray, 3 judges suggested international terrorism concern

enough. External affairs and federalism

o Poses significant threat to federal balance. Gibbs CJ’s dissents in Koowarta and Tasmanian Dam.

o Signs that HC is becoming more cautious, such as in Industrial Relations Act Case.

Week 6: Defence Power Commonwealth under this power has been able to legislate about a lot of State matters- to

that extent it is a concurrent power. But specifically regarding military- effectively an exclusive power given s 114.

s 68- GG is commander in chief, acts on advice of executive. s 119- Cth has to protect States against invasion, and when help is requested. Power is purposive: Stenhouse v Coleman per Dixon J. Is this law for a defensive purpose? Power is elastic- scope expands depending on historical and social circumstances.

Peacetime. Wartime. Transition. Much more broad in wartime. Communist Party Case (1951)- Fullagar J noted two aspects:

o Primary- laws that have defence as their direct and immediate object. Peacetime and wartime.

o Secondary- range of matters only indirectly related to defence. Wartime only. Wartime power

o Extremely broad. See Farey v Burvett (1916), where bread price-fixing legislation was upheld.

Really concerns internal trade and commerce, but purpose was assisting war effort by dealing with wartime challenges.

Several judges would not be supported by defence power in peacetime.o Lloyd v Wallach (1915)- upheld legislation allowing minister to detain anyone who

was a threat. Power extends to preventing acts that would undermine the war effort. Contrast Communist Party Case (peacetime).

o Series of WWII decisions upheld price controls, rent controls, and employment regulations. Supported by need to tightly regulate economy during wartime for security and to support war effort, and to maintain social stability.

13

Page 14: · Web viewoncerned ‘tax bonus payments’ made by Rudd government. Argued that payments were unsupported by a head of power. Upheld legislation as a valid exercise of s 61, together

UQ Semester 1, 2014 Defence and taxation

o Commonwealth seized control of income tax in 1942: (a) dramatic increases in federal income tax levels (b) grants to states equal their tax revenue conditional on repeal of taxes (c) rule requiring pay federal tax before state tax (d) compulsory acquisition of state tax office staff and facilities

o Challenged in First Uniform Tax Case (1942). (a) and (c) authorized by taxation power. (b) authorized by s 96. (d) authorized by defence power.

o Second Uniform Tax Case (1957) (c) and (d) did not survive, but by then it was too late.

Limits on wartime powero R v University of Sydney; Ex parte Drummond (1943)- no attempt to encourage

excluded students to contribute to war. Law lacked sufficient connection to defence. Need link to defensive purpose.

o Industrial Lighting Case (1943)- restrictions on lighting for industrial premises. Law lacked sufficient connection to defence. Only related to defence insofar as any provisions for public good are related to defence during wartime. Have to show something more than public good- that this is related to defensive purpose.

o Jehova’s Witnesses Case (1943)- restrictions unsupported by defence power, as they could be used for reasons unrelated to defence. Regulations overly broad. JW also had other views.

Transition to peaceo Power’s scope is less than wartime, but more then peacetime. But ceases to

operate after certain period. Reasonable time limit.o Dawson v Commonwealth (1946)- post-war challenge to restrictions on sale of land.

Dixon J accepted defence power must extend to sustaining for some reasonable period of time measures necessary for the transition to peace. Economic and social challenges don’t all immediately end as soon as peace agreement reached.

Same result in series of post WWII cases on preferential housing and employment for returning service people.

o R v Foster; Ex parte Rural Bank of NSW (1949)- regulations struck down. Defence power does not cover any problem created/aggravated by war- would potentially cover anything.

Peacetime powero Clothing Factory Case (1935)- upheld legislation establishing factory to produce

military and civilian clothes.o Capital Issues Case (1953)- upheld legislation allowing Treasurer to restrict

businesses from raising capital in ways that competed with government capital issues.

Argued necessary to protect revenue stream government might have to use in wartime.

Distinguished Communist Party Case.

14

Page 15: · Web viewoncerned ‘tax bonus payments’ made by Rudd government. Argued that payments were unsupported by a head of power. Upheld legislation as a valid exercise of s 61, together

UQ Semester 1, 2014o Shipping Board Case (1926)- authorizing shipping board to produce and sell

equipment for generating electricity. Argued electricity needed for defence. Struck down. Not a sufficient connection to defence. Law is primarily for purposes other than defence.

Might be some defensive benefit but connection not tight enough. Might be upheld in wartime. Distinguish University of Sydney?

o Communist Party Case (1951)- struck down. Judges rejected argument that international context justified legislation. Dixon J- no opinion of Parliament as to some matter or event that would relate the subject of law with power can suffice to give the law that relation. HC itself has to look.

Also emphasis on rule of law. Law targeting specific person/group less likely to be supported by defence power than generally phrased legislation (discretion with judges).

May be permitted in wartime- see Lloyd. Military discipline

o Defence power supports establishment of service tribunals to hear charges against military personnel. See R v Tracey; Ex parte Ryan (1989).

o But Cth cannot prevent civilian courts from also trying military personnel. Internal threats

o HC generally confined defence power to external threats.o Police power usually arises from incidental power (s 51(xxxix))and executive power

(s 61).o Some suggestions of using defence power against internal threats: Thomas v

Mowbray. Power extends to holding off violent attacks against bodies politic of Cth and

States. Did not regard defence power as covering general policing.

Week 7: Corporations Power Section 51(xx). Concurrent power over foreign corporations, and Australian trading & financial

corporations. Interpreting the power

o Massive source of power for the Cth to regulate economic activity.15

Page 16: · Web viewoncerned ‘tax bonus payments’ made by Rudd government. Argued that payments were unsupported by a head of power. Upheld legislation as a valid exercise of s 61, together

UQ Semester 1, 2014o Interpretation raises two primary issues:

(1) What are the entities with respect to which Parliament may legislate? (2) What kinds of legislation may Parliament make with respect to these entities?

What is a corporation?o Association of people given artificial legal personalityo Private companies become corporations by being registered under Corporations Act or

other statuteo Local government authorities are also typically incorporated under legislationo Company has separate legal entity, shareholder enjoy limited liability, perpetual

succession. Foreign Corporations (all, not just trading or financial)

o Any corporation not formed within Cth. Look at foreign law to see when such a corporation is created.

o Regarded as a corporation if, under foreign law, has separate legal personality from its members

Trading Corporationso Corporation set up primarily to engage in trade – 3 testso Test 1: Purpose of creation

The nature of the corporation is determined by the purpose for which it was created

Involves looking at what a corporation actually does R v Trade Practices Tribunal; Ex parte St George County Council (1974)

Council supplied electricity to consumers as part of local government functions. Was the council, a body corporate established under the Local Government Act 1919 (NSW), a trading corporation?

Held: no. Despite fact that they clearly engaged in a trading activity, majority applied the purpose test and said that corporation has been formed for the purpose of administering local government rather than trading.

Gibbs CJ : council was set up not to trade, but ‘for the purposes of local government to provide an essential service to the inhabitants’ in the way ‘most beneficial to them’.

Menzies J : companies established to undertake public works for community benefit may not be trading corporations for constitutional purposes.

Clearly flawed- enables trading corporations to avoid regulation by stating other purpose.

o Test 2: Predominate activity Looks at the current activities of the corporation Adopted by the two dissenting judges, Barwick CJ & Stephen J, in St George

County Council. Barwick CJ : trading corporation is a corporation whose ‘predominant and

characteristic activity’ is trading in goods or services. Purposes for which corporation trades irrelevant. Company’s current

activity matters.o Test 3: Substantial activity

16

Page 17: · Web viewoncerned ‘tax bonus payments’ made by Rudd government. Argued that payments were unsupported by a head of power. Upheld legislation as a valid exercise of s 61, together

UQ Semester 1, 2014 Whether trading activities are ‘substantial’ or ‘not insignificant’ part of

company’s operations. Shortcomings of purpose test led to reconsideration of what constituted a

trading or financial corporation in Adamson’s Case. This test adopted by Mason and Murphy JJ.

Adamson’s Case (1979) AFL player argued that a term of his contract, contravening him from

moving from one club to another contravened provisions of the TPA- argument depended on finding that the AFL and its clubs were trading or financial corporations.

Majority rejected approach in St George, holding instead that it was the activities a corporation rather than the purpose for which it was formed that would determine whether it was a trading or financial corporation.

AFL/its clubs were trading corporations because they engaged in significant trading activities such as selling of TV broadcast rights football leagues not set up primarily to trade, but trade was substantial part of their day to day operations.

Irrelevant that AFL/its clubs were incorporated under legislation designed for non-trading corporations and they were non-profit organisations that put any surplus income back into the sports they administered. The fact that they derived significant income from trading activities sufficed to define them as trading corporations.

Mason J : trading activity must form ‘a sufficiently significant proportion of its activities’.

Tasmanian Dam (1983) Held that a corporation would be a trading corporation if trading was a

substantial part of its activities, ‘substantial’ being defined as ‘not insubstantial, tenuous or distant’.

Corporation would be considered trading corporation even if trading was not primary or dominant. Terms ‘substantial’ and ‘significant’ include activities that amount to less than 50% of activities. Precisely how small a % it can be has not been stated.

The substantial activity test seems to be most favoured by the HC today. The Purpose Test and Shelf Companies

o Although somewhat redundant can still be used where activities test doesn’t work e.g. shelf companies

o Shelf companies –formed by lawyers to sell them to clients who want to incorporate their business quickly without having to go through the process of registering the company themselves

o Fencott v Muller (1983) Considered whether shelf company (no trading activities yet) was a

constitutional corporation. Purpose (in constitution) indicated that once active would engage in trading

activities.o Held that the purpose sufficed to make it a trading corporation.

Seems that corporation will be covered if it satisfies any of purpose/activities tests. Intra-state trading corporations

17

Page 18: · Web viewoncerned ‘tax bonus payments’ made by Rudd government. Argued that payments were unsupported by a head of power. Upheld legislation as a valid exercise of s 61, together

UQ Semester 1, 2014o Early HC s 51(xx) does not cover exclusively intra-state trading corporations (based

partly on reserved powers doctrine: Huddart Parker).o Overruled in Concrete Pipes Case. Held Huddart Parker was wrongly decided on

reserved powers.o Decision opened way for Cth to use power as major tool to regulate the economy,

because by regulating business activities of corporations, was able to regulate almost all economic activity.

Financial Corporationso Re Ku-ring-gai- Deane J- Corporation that engages in commercial dealing in finance.o Definition was adopted by Mason, Murphy and Deane JJ in the Superannuation Board

Case (1982)o Company fulfils definition when it trades in finance as opposed to dealing in

goods/services.o State Superannuation Board v Trade Practices Commission (1982)

Whether superfund for Victorian public servants was a financial corporation. Majority held that it was because financial activities constituted a substantial part of its operations (Deane J)

Gibbs CJ and Wilson J (in dissent) argued purpose of corp. was governmental not financial

Scope of the Powero Power over a particular type of entity, not purposive power or power to regulate a

specific activity.o HC determines outer limits on case-by-case basis. Decided cases reveal some general

principles:o (1) Creation and Abolition

Cth lacks the power to legislate with respect to the incorporation of corporations. Words ‘formed within the Cth’ refer to corporations already formed under state laws (New South Wales v Cth (Incorporation Case) (1990))

Parliament may regulate the activities of corporations, but not ban them or remove their legal status (Cth v Bank of New South Wales (Banking Case) (1948))

Cth may sometimes incorporate corporations by relying on other heads of power (eg ABC).

See for example ANA v Commonwealth.o (2) Distinctive Character Test

Some HC decisions have supported view that law will only fall within corporations power if subject matter of law relates to ‘distinctive character’ of trading and financial corporations.

Law would have to relate to some aspect of trade or finance activities. Re Dingjan; Ex parte Wagner (1995)- ruling that legislation enabling a tribunal to

set aside ‘unfair contracts’ entered into by constitutional corporations was outside the power

Tasmanian Dam (per Deane J) – trading and non-trading activities can’t always be separated

o (3) Object of Command Test- dominant test Whether the object of statutory command is a constitutional corporation.

18

Page 19: · Web viewoncerned ‘tax bonus payments’ made by Rudd government. Argued that payments were unsupported by a head of power. Upheld legislation as a valid exercise of s 61, together

UQ Semester 1, 2014 If law instructs constitutional corporation to do/not do something, likely to fall

within s 51(xx). Provision also supports laws aimed at protecting corporations from conduct

intended or likely to cause them loss or damage.o (4) Activities Done by Third Parties

Power held to support laws regulating what third parties do to corporations. Actors and Announcers Equity v Fontana Films (1982)

Concerned TPA provision prohibiting secondary boycotts of corporations- where action taken against corporation to stop it dealing with a third party who is real target.

Provision upheld on basis that it regulates third party actions that are intended to cause the corporation’s loss or damage.

The boycott amounted to an interference with a contractual relationship of the corporation, and thus was something done to the corporation.

Is a limit to how far the courts are willing to go in recognising that conduct engaged in by someone other than a corporation is a law ‘with respect to’ a corporation

Re Dingjan; Ex parte Wagner- where a corporation has contract with A and A has contract with B (neither corporations), power does not extend to regulating the relationship between A and B, even if that relationship has an economic effect on the corporation. NB: overruled by Concrete Pipes

o (5) Activities Preparatory to Trading and Financial Activities Tasmanian Dam- not only the trading/financial activities that could be regulated

but also activities that were only preparatory to trading or financial activities.o (6) Activities by Human Agents of Corporations

R v Australian Industrial Court (1977)- held implied incidental power to s 51(xx) included power to regulate conduct of human agents and thus Cth could validly criminalise directors for being involved in offences by corporations.

Work Choices Case (2006)- most important recent decision on corporations powero Work Choices legislation was series of controversial amendments to Workplace

Relations Act 1996.o WRA supported by s 51(xxxv) [conciliation/arbitration power]. o However amendments sought to rely on s 51(xx) and regulate single-state employers.o Legislation was challenged on a number of grounds, including that:

Deals with internal relations of corporations rather than relations with external entities- argument was rejected as unjustifiably limiting the plain words of the power.

Specific industrial relations power precludes using s 51(xx) for industrial relations purposes- rejected based on the Engineers doctrine and multiple characterisation.

o The majority judges upheld legislation based on a broad reading of s 51(xx). The law regulates constitutional corporations and is therefore within the power.

o Strong dissenting judgments from Callinan and Kirby JJ- unless limits are placed on powers like s 51(xx) the federal balance will be destroyed.

19

Page 20: · Web viewoncerned ‘tax bonus payments’ made by Rudd government. Argued that payments were unsupported by a head of power. Upheld legislation as a valid exercise of s 61, together

UQ Semester 1, 2014o Broad formulation true to say that the Cth can use s 51(xx) to regulate any aspect of

corporate activity, any relationship that a corporation has and any conduct of others that affects corporation

Limits of the Powero HC has occasionally been willing to strike down law for not being sufficiently connected

with power.o Davis v Cth (1988)

Legislation establishing Australian Bicentennial Authority (ABA) as a corporation. The law sought to give the ABA a monopoly over certain words and symbols, including ‘200 years’.

HC held that the corporations power would cover prohibitions on misleading use of corporate names or symbols, but granting a monopoly over ‘200 years’ was beyond its scope.

But arguably open to challenge after Work Choices?

20

Page 21: · Web viewoncerned ‘tax bonus payments’ made by Rudd government. Argued that payments were unsupported by a head of power. Upheld legislation as a valid exercise of s 61, together

UQ Semester 1, 2014

Week 8: Taxation, Appropriation and Spending Magna Carta & Bill of Rights taxation requires consent of governed. No express provision in

Constitution, but assumes people’s rights cannot be taken away except under the authority of law.

Judicial confirmation of this in Deputy Federal Commissioner of Taxation v Truhold Benefit. One way of controlling government power clear/transparent rules on raising and spending

revenue. Principle- No taxation except under authority/law of Parliament. Powers

o s 51(ii)- concurrent. Necessary implication of federal division of powers that Commonwealth only has power over taxation for its own purposes. Legal conflict between Commonwealth and State tax laws seen as an impossibility: Victoria v Commonwealth.

o s 90 - power to impose duties of customs and excise - exclusive powero s 51(i) - power over trade and commerce (next topic)

Procedural limits (to ensure accountability):o s 81- CRF- one pool to prevent siphoning off before money gets thereo s 83- withdrawal by law (Appropriation Act) onlyo s 99- can’t give preference in laws of taxation/revenueo s 53-

Taxation/appropriation bills may not originate in Senate. Taxation/supply bills may not be amended by Senate. But they can reject and

return with amendment request. Other appropriations bills may be amended by Senate, but not to increase charge/burden on people.

Old English compromise- Lords would not block money bills if Commons did not tack on substantive law.

Constitutional convention- government must resign when defeated on a supply bill.

o s 55- 1) tax law must deal only with tax (no tacking on) (Matthews v Chicory MB, Air

Caledonie) 2) (other than customs/excise) only one subject of taxation only. Customs/excise

laws shall deal only with customs/excise respectively. (Air Caledonie (tax), Second Fringe Benefits Tax Case (custom))

Convention- upper house will not block supply bills.o s 54- OASG/supply bills must deal only with OASGo s 51(ii)- can’t discriminate between States/parts of Stateso s 88- duties of customs and excise to be uniform throughout the Commonwalth.

Is the charge a tax? Matthews v Chicory Marketing Board (Latham CJ):o 1. A compulsory exaction of money;o 2. By a public authority (Australian Tape Manufacturers Association Ltd v Cth);

21

Page 22: · Web viewoncerned ‘tax bonus payments’ made by Rudd government. Argued that payments were unsupported by a head of power. Upheld legislation as a valid exercise of s 61, together

UQ Semester 1, 2014o 3. For public purposes (Tape Case, Air Caledonie); o 4. Not a payment for services (Air Caledonie, Air Sevices v Canadian Airlines) o Fines/financial penalties for breaches of law are not taxes (s 53): MacCormick v

Commissioner of Taxation Compulsory Exaction or Charge

o Person has no choice but to pay the charge- sufficient to meet the requirement of ‘compulsory’ (Victoria v Commonlwealth ) (the Payroll Tax case)

o Charge regarded as compulsory even if statutory scheme offers alternative to payment, if alternative payment is a burden which the taxpayer would naturally seek to avoid (Dixon J: A-G (NSW) v Homebush Flour Mills)

Compulsory acquisition scheme- state expropriated flour from millers, who were then given compensation and a first option to buy their flour back at a higher price. Millers required to store flour until sold by state.

Challenge: tax was an excise duty which can only be levied under s 90 by Commonwealth. Held (Dixon J): essentially same as if exacted by means of sanctions designed to that end.

By a public authority o Rule has been questioned. Majority (Mason CJ, Brennan, Deane and Gaudron JJ) in Tape

Manufacturers suggested that not essential to concept of a tax that exaction should be by a public authority (contrast this to position taken in Matthews v Chicory Marketing Board)

o Majority held: whether this step should be taken depends upon what meaning would be given to the expression ‘non-public’ authority if one of its functions is to levy/demand/receive exactions’ to be expended on public purposes. Not necessary to decide question for outcome of case.

For a public purpose o Logan Downs v Commissioner of Taxation- HC conceded that requirement practically

impossible to police.o Luton v Lessels- exaction of money to enforce an existing legal obligation is not a tax.

Fees for services rendered o Air Caledonie v Commonwealth (1998) [definition and also s 55] (Unanimous judgment)

Amendment to Migration Act- $5 from all entering passengers, charge on airline (whether or not they collected from passengers). If tax, could not be in Migration Act under s 55. Held: tax.

Court held that the fee had all the positive attributes which have been accepted as prima facie sufficient to stamp an exaction of money with the character of tax: compulsory (unrealistic to argue that people entering had a choice to pay), exacted by public authority (the Commonwealth itself) for public purposes (Consolidated Revenue s 81) and it (or its amount) was enforceable by law.

Key issue: was it a payment for services?- licensing passenger to enter. HC rejected argument- fee applied to returning citizens, who did not need special license to re-enter as they had right to re-enter. While it may be necessary to subject citizens to administrative procedures when re-entering, requiring citizen to submit to that inconvenience could not properly be seen as providing services to or at request of citizen concerned.

22

Page 23: · Web viewoncerned ‘tax bonus payments’ made by Rudd government. Argued that payments were unsupported by a head of power. Upheld legislation as a valid exercise of s 61, together

UQ Semester 1, 2014 In order to be payment for service, must be an identifiable service rendered.

Also, fee must be directly related to the provision of the service. If first hurdle had been discharged, issue would have arisen under second- fee paid by airlines and not passengers.

s 55 strictly applied- although amending legislation was tax only, Migration Act dealt with other matters.

o Airservices Australia v Canadian Airlines Compass (liquidated) leased from CA. Compass owed Airservices (statutory,

associated with government authority regulating air traffic services. Airservices imposed fees (statutory lien) on CA.

Fees were for public safety functions. Fixed according to complex formula. However- did not cost actual service to each airline. Compass charged more than value of service. Held: payment for services.

Not necessary to demonstrate charges directly correspond to value to user of privilege or cost of providing privilege. Often the case when a statutory corporation (Airservices) was set up with a power to provide services on a user-pas basis. Validity of the law is determined by reference to its operation, not by reference to parliamentary objectives.

Fees imposed not to raise revenue but to cover cost of services across a group of users. Fee reasonably related to expenses incurred (in providing services).

o See also: Harper v Minister for Sea Fisheries; Hematite Petroleum Pty Ltd v Victoriao Qureshi v Minister for Immigration and Multicultural and Indigenous Affairs- Request for

service not essential, as long as it is provided. Kenny J held recovery of cost of maintaining an illegal immigrant in detention against his wishes was not a tax. Charge could not be properly seen as a fee for services as they were clearly not desired by the immigrant. When the purpose is locking him up, can’t be seen as a service to the detainee, or at his/her direction. Nevertheless, still not a tax as its purpose was not to raise revenue, but to recoup costs. Best regarded as a condition of the detainee’s presence in Australia.

One subject rule (s 55)o Second Fringe Benefits Tax Case: Act imposed taxes on several categories of fringe

benefits. Majority: question of whether statute relates to one or more subjects of taxation involves an assessment of both fact and value. Deferred to Parliament’s understanding that law was all on one subject. Common subject: fringe benefits. Court will generally not override Parliament’s view (presumption) unless matters clearly separate/distinct.

o Purpose of the second paragraph of s 55 was to prevent tacking together of tax bills of different kinds and unlimited number in one measure- non-compliance with this would result in complete activity.

Non-discrimination rule- s 51(ii)o Fact that uniform tax impacts differently owing to local conditions does not make it

discriminatory (Cameron). o R v Barger (1908) [keep in mind influenced by now discarded doctrine of reserved state

powers]: Statutory Excise Tariff provided that manufacturers providing ‘fair and reasonable’ labour conditions were exempted from excise duty.

23

Page 24: · Web viewoncerned ‘tax bonus payments’ made by Rudd government. Argued that payments were unsupported by a head of power. Upheld legislation as a valid exercise of s 61, together

UQ Semester 1, 2014 Griffth CJ/Barton/O’Connor JJ- purpose of Act, apparent on face, whatever

attempt might be made to disguise it in title, not make revenue, but regulate labour conditions - substance not form.

Some states had industrial awards clearly regarded as ‘unreasonable’ by the Commonwealth. The Commonwealth may not impose discriminatory taxation to enforce its preferred policies.

Held provision was discriminatory, as it was clearly intended to allow the imposition of different duties in different states. Rejected argument that if there was any discrimination, it operated by reference to the employment/economic characteristics of different localities not their state characteristics.

o Elliot v Commonwealth (1935) - contrast Barger s 99- no discrimination between states in laws of trade/commerce/revenue.

Same standard as 51(ii). Maritime workers at prescribed ports required licence to work in those ports. The

Commonwealth designated ‘prescribed ports’ in all states except Western Australia and Tasmania.

Latham CJ : marked difference in words of s 51(ii) and s 99 - expressly distinguishes between preferences to States and preference to parts of States ie. Sydney v Newcastle, not whole of NSW.

Dixon J [dissent]: in specifying chief ports in each of four states, a course was taken which must be considered as affecting each of those States as a whole - regulations gave a commercial advantage to the states where the prescribed ports were located and that this was contrary to s 99.

Evatt J : s 99 may apply although legislation/regulations contain no mention of State- eg the section may be infringed if preference is given to part of state over another State/part of another State.

Rich, Starke, McTiernan JJ : no preference within s 99. Majority upheld the law. Discrimination not between states, but rather between

different Commonwealth ports, having regard to local conditions.o Distinction

Barger legislation clearly intended to authorise discrimination between the states.

Elliott legislation sought to apply uniform conditions to Commonwealth ports, regardless of location. Conditions just happened to be relevant to ports in some states and not others. Discrimination incidental

Regulatory use of taxationo Fairfax v Federal Commissioner of Taxation (1965)- did not actually overrule Barger

Income from superannuation funds taxed unless invested in prescribed public securities. Purpose not to raise funds, but to give employer’s incentive to invest in public sector.

Challenge: outside taxation power as purpose unrelated to revenue (No super funds power). Rejected (unanimous). Can use power to regulate activities outside jurisdiction (multiple characterization)

Form and substance- subject matter to be determined by reference solely to the operation which the enactment has if it be valid- reference to nature of rights, duties, powers and privileges. Discussed Barger, made reference to the fact that it was decided with respect to doctrine of reserved powers

24

Page 25: · Web viewoncerned ‘tax bonus payments’ made by Rudd government. Argued that payments were unsupported by a head of power. Upheld legislation as a valid exercise of s 61, together

UQ Semester 1, 2014 Substance of enactment is imposed obligation. Only obligation pay income tax

under s 51(ii) Duties of customs and excise- exclusive to Cth (s 90)

o Customs duties- duties on imports/exports. Excise duties- levies on goods at any point of production/distribution process as they move from manufacturer to consumer.

o Charge typically only identified as excise duty if connected to output (charge reflects amount produced)

o The states may impose consumption (on consumer at point of sale) taxes: Dickenson’s Arcade v Tasmania (1974)

o Peterswald v Bartley (1904)- license fee on brewers not an excise duty, as it was not related to output.

o Matthews v Chicory MB (1938)- Levy on chicory growers calculated by acreage planted. Held to be sufficiently related to output to qualify as excise duty, notwithstanding that tax unrelated to quantity/value produced.

o ‘Excise- tax on or connected with commodities’, would be incorrect to confine it to taxes which had an arithmetical relation to quantity or value.

o Parton v Milkboard- not limited to taxes on goods while they are in the hands of the manufacturer but may apply to goods at all points in the chain of distribution until it reaches the consumer.

o Distinction has arisen between license fees and duties of excise. License fee levied on retailers/consumers at the point of sale is not an excise duty, even if based on turnover: Dennis Hotels v Victoria (1960).

o However, a tax on retailers based on the production or distribution of the goods, rather than purely their sale, is an excise duty: Capital Duplicators v Australian Capital Territory (No 2) (1993).

Appropriation and spendingo All funds raised by Commonwealth CRF (s 81). Funds can then be appropriated for

the purposes of the Cth.o Appropriation from CRF must be authorised by legislation: s 83.o ‘Purposes of the Commonwealth’ not confined to subjects listed in s 51 HC

interpreted the term widely.o AAP Case (1975)

AAP made tied grants to regional councils, conditional on implementation of Cth government policies.

Challenged on basis that funds not being used for the ‘purposes of the Commonwealth’. Rather, they were being used for local government activities.

Held: ‘purposes of the Commonwealth’ are whatever purposes Parliament chooses.

By giving tied grants to states/local councils, Cth may accomplish policy objectives outside powers.

May also give tied grants to states (s 96) if they implement policy- Second Uniform Tax Case (1957).

o Combet v Commonwealth (2005) 224 CLR 494 - how specific an appropriation must be to fall within s 81

25

Page 26: · Web viewoncerned ‘tax bonus payments’ made by Rudd government. Argued that payments were unsupported by a head of power. Upheld legislation as a valid exercise of s 61, together

UQ Semester 1, 2014 Argued that general appropriation in supply bills for ‘departmental expenditure’

did not cover expenditure on advertising campaign not mentioned in budget notes.

Held: upheld validity of expenditure. Parliament may decide how precisely to express appropriations- ‘One-line appropriations are valid’ (Gleeson CJ). McHugh and Kirby JJ dissented, arguing that appropriations must be precise enough to be meaningfully evaluated.

o Pape v Commissioner of Taxation (2009)- qualified the broad approach to appropriation in the AAP Case

Concerned ‘tax bonus payments’ made by Rudd government. Argued that payments were unsupported by a head of power. Upheld legislation as a valid exercise of s 61, together with s 51(xxxix).

Neither ss 81 or 83, nor a combination of two confer a source of a ‘spending’/ ‘appropriations’ power

HC accepted that s 81 only covers spending that relates to Commonwealth purposes as defined elsewhere in Constitution. Includes s 51 and s 61. Covered by s 61 implied nationhood power.

May look to executive power of Commonwealth (s 61), subject to appropriation requirement and supportable by legislation made under the incidental power in s 51(xxxix).

French CJ found it unnecessary to consider the validity of the Act under the taxation power.

o Williams v Commonwealth [2012] HCA 23 - recent appropriations case Cth executive entered into funding agreement with Scripture Union Queensland

to provide chaplaincy services at a state school. Challenged validity as arrangement not given effect by a specific Commonwealth law. Commonwealth submitted that the provision could be given effect as a law enacted under the Corporations power.

Held: the law was beyond the executive power conferred by s 61. Relationship between s 61 and s 96 - if s 61 includes a power to spend any money lawfully appropriate, regardless of the purposes for which or circumstances in which the expenditure is to be applied, several consequences would follow to suggest the proposition is flawed. Was no statute supporting the agreement. The Commonwealth executive has only very limited power to spend money without parliamentary authorisation. Heydon J held in dissent that the executive could spend money on anything within the Commonwealth’s legislative powers.

Week 9: Freedom of Interstate Trade Relevant to Australian federation- association of semi-autonomous units who all commit to

common market. To ensure economic prosperity, remove trade difficulties/barriers, strengthen whole region’s

economy. VFI- problem in Australia- lack of revenue streams for the states- centralisation of taxation-

can’t meet their obligations without federal assistance- susceptible to federal influence HFI – not a problem in Australia – federal government redistributes among states accordingly.

But tensions? Conditions that help federations maintain economic health:

26

Page 27: · Web viewoncerned ‘tax bonus payments’ made by Rudd government. Argued that payments were unsupported by a head of power. Upheld legislation as a valid exercise of s 61, together

UQ Semester 1, 2014o Flexible economy - free movement of; goods, services, capital, peopleo Stable and clear centralised rules relating to international and interstate tradeo Clear rules in each state relating to intrastate trade, with possibility of regulatory

competitiono Minimising fiscal imbalances

Pre-federation, colonies were highly protectionist, caused ongoing tensions that persisted after federation?

o Constitutional solutions: s 51(i), s 90, s 92 (including but really useless – s98, 99, 102). Section 92

o Strongly worded- “absolutely free”. But not an absolute freedom. Subject to legal limits, but framers may have intended these limits to be minimized and uniform throughout the country.

Free Trade theoryo Early cases focused only on discriminatory barriers that distinguished internal and

external traderso Fox v Robbins- struck down WA law imposing higher license fees on bars selling

imported beer. Individual rights theory

o James v SA- challenged SA Dried Fruits Board who ordered him not to export produce. Did not discriminate between internal and external producers, but HC agreed nonetheless infringed s 92. In Fox, was treating external traders worse- protectionist. But here, law aimed to make him worse off- not protectionist or discriminatory. Individual rights- focusing on reasonableness of barriers or restrictions in individual rights.

o State Minister for Agriculture then compulsorily acquired James’s crop under legislation so they could decide where it went. James challenged this and won again.

o Commonwealth sought to acquire James’s produce. James challenged law before PC and won.

o See also Banking Case: Commonwealth sought to nationalize banking industry, creating monopoly. Held to violate s 92 even though not discriminatory. A monopoly is only justified if it is the only reasonable mode of regulation [of that industry].

Cole v Whitfield (1988)- free trade theoryo HC decided to clarify law in a unanimous joint judgment. Opted for free trade theory.o Concerned regulations prohibiting taking/trading/possession of undersized crayfish in

Tasmania.o Law was mainly a conservative measure- did not distinguish between external/internal

traders.o Whitfield imported undersize crayfish from SA (where no such prohibition existed).

Charged.o Higher burden on Tasmanian farmer- so not protectionist.o Held: does not burden interstate trade. Regulations reasonable- no practical way of

distinguishing between local and imported crayfish. ‘Proportionate to a legitimate objective’. s 92 only ban ‘discriminatory burdens of a protectionist kind’.

Cole v Whitfield Testo 1. A burden on trade [typically not difficult to satisfy]o 2. Discrimination between internal and external traders.

27

Page 28: · Web viewoncerned ‘tax bonus payments’ made by Rudd government. Argued that payments were unsupported by a head of power. Upheld legislation as a valid exercise of s 61, together

UQ Semester 1, 2014o 3. A protectionist impact.o Also supplied a non-exhaustive list of four types of laws which would violate s 92:

Tariffs that increase the price of imports Quotas on imports Differential railway rates Subsidies for local goods

Bath v Alstono Applied above test. Higher license fees imposed on sellers of imported tobacco. Victoria

argued merely leveling playing field, as external tobacco wholesalers did not have to pay Victoria license fee for wholesalers.

o Law struck down. Had protectionist effect of disadvantaging imported tobacco. Current interpretation of s 92 also affirmed in Betfair v Racing NSW (2012). Castlemaine Tooheys v SA

o Non-refillable bottles required a compulsory refund of 15c + returned to the retailer. Refillable bottles required a compulsory refund of 4c + returned to the collection depots [easier].

o Local beer mainly sold in refillable bottles Imported sold in non-refillableso Not discriminatory prima facie but violated s 92.o Protectionist and discriminatory because it Severely limited Toohey’s market share

[discouraged retailers from selling outside bottles].o The stated aims were environmental – this did not require two different methods of

returning bottles.o Don’t have to have an explicitly discriminatory law for it to have a discriminatory

burden under s 92 o Joint judgement (Mason CJ, Brennan, Deane, Dawson and Toohey JJ)

Law burdening interstate trade will only be consistent with s 92 if it is necessary or appropriate and adapted to a legitimate policy objective

The burden placed on interstate trade must be incidental to the law. A law that deliberately burdens interstate trade will be invalid.

The burden must not be disproportionate. A discriminatory law will only be upheld if there is no other reasonable

alternative. Betfair v WA (2008)

o WA prohibited online betting to protect integrity of betting industry. Local betting dominated by TAB

o Had discriminatory impact, as local WA betting was dominated by TAB. P’s were Tasmanian.

o Prima facie not discriminatory. Found non-discriminatory objective Wasn’t enougho Following Castlemaine also have to show its disproportionate- banning not only way to

regulateo Discretion also given to executive to make exceptions. Don’t know grounds of

exceptions. Not enough to mitigate discriminatory nature.o Discriminatory prohibition invalid if less drastic option available to achieve the same

policy objective Betfair v Racing NSW (2012)

28

Page 29: · Web viewoncerned ‘tax bonus payments’ made by Rudd government. Argued that payments were unsupported by a head of power. Upheld legislation as a valid exercise of s 61, together

UQ Semester 1, 2014o Legislation imposed fee for bookmakers to use information about races in NSW. Argued

that impacted more heavily on online bookmakers. HC upheld legislation because imposed uniformly on intrastate and interstate bookmakers – not proven to have protectionist effect.

State monopolieso Early cases such as James & Banking Case suggested that government monopolies

would offend s 92.o Barley Board Case (1990)

Upheld scheme where all barley grown in NSW had to be sold through BMB. HC considered scheme not discriminatory and therefore consistent with the free

trade theory [disadvantaged instead of advantaged NSW traders]. James likely bad law.

Did note that where a commodity was a scarce resource local to a particular state, the imposition of a government monopoly might offend s 92. Would capture market for good across whole nation- distorts national economy to that extent- might offend s 92. Would be an extension of Cole v Whitfield. Arguably would be protectionist for the State with resource.

Banking Case might still be decided the same way today?o Joint judgment in Betfair v WA pointed out that local and interstate commerce are

interrelated. State based restrictions on a national market may therefore undermine the spirit of s 92.

Freedom of Intercourse – s 92 “intercourse among the States”o Cole v Whitfield – confers ‘a personal freedom to pass to and fro among the States

without burden’o Nationwide News v Wills (1992)

Deane and Toohey JJ – freedom may cover not only physical movement, but ‘all of the modern forms of inter-state communication’.

Brennan J – discrimination\protectionism is not a necessary element of the test for freedom of intercourse. A restriction on border crossing between the states will be invalid unless:

The law is mainly enacted for a purpose other than limiting interstate movement

The burden on movement is appropriate and adapted to the intended purpose

The limitation on movement is necessary and incidental to the law’s operation.

It is unclear whether the freedom extends to movement of information (as Deane and Toohey JJ suggested) or only persons and goods.

1. Does the law impose a burden on interstate trade?

Is some restriction placed upon trade that occurs interstate?2. Is the burden discriminatory

Castelmaine Tooheys v SAo Can be discriminatory in practice if it “practically” places heavier burden on

interstate traders29

Page 30: · Web viewoncerned ‘tax bonus payments’ made by Rudd government. Argued that payments were unsupported by a head of power. Upheld legislation as a valid exercise of s 61, together

UQ Semester 1, 20143. Does the law have a protectionist effect?

If the law confers a market/competitive advantage on the local goods insofar as it requires interstate competitors to comply with local regulations

4. Alternatively is the law a proportionate measure in pursuit of a legitimate objective?

Look at the stated objective and see if it is legitimate – Castlemaine Tooheys – Promote recycling – legitimate concern of government

Is the pursuit of the objective proportionalo Betfair v WA

Court found that the regulation of the exchanges had alternative means available which did achieved the same objective (protecting the integrity of racing) while placing fewer restrictions on trade

Where alternative means are available which place fewer restrictions on interstate trade for the same objective – law disproportionate to objective and therefore invalid – different bottle returning procedure in Castlemaine Tooheys

30

Page 31: · Web viewoncerned ‘tax bonus payments’ made by Rudd government. Argued that payments were unsupported by a head of power. Upheld legislation as a valid exercise of s 61, together

UQ Semester 1, 2014

Week 10: Express Constitutional Rights Right- claim that somebody else do or not do something. Entails correlative duty on other

party. Freedom need not involve any correlative duty. However freedoms need legal protection to

be meaningful. This protection is typically provided by enforceable rights. Immunity is a right that someone not exercise a legal power over me. Other party has duty

not to exercise power. Right not to be interfered with in particular ways. Common law freedom. Assumed legislation does not intend to encroach unless it clearly

manifests an intention to do so. Legislative rights arise from statutes/subordinate legislation. But subject to modification by

case law/statutes. Constitutional rights- cannot be overridden by judiciary or legislature. Often immunities

against government action. Can only be removed by constitutional amendment (though scope clarified by judicial

interpretation). Express Constitutional Rights

o s 92- right to freedom of interstate trade, commerce and intercourse.o s 117- right not to be discriminated against on grounds of residency.o s 51(xxxi)- right to compensation for any acquisition of your property by

Commonwealth.o s 80- right to trial by jury when indicted for a Commonwealth offence.o s 116- right to freedom of religion.o s 41- right of state electors to vote in Commonwealth elections?

Acquisition of propertyo Right applies only to Cth. No guarantee of compensation for property taken by States-

Pye v Renshaw. o States have legislation providing for compensation, but this can be taken away by

ordinary statute.o HC- broad interpretation of property and narrow interpretation of acquisition.o Property includes:

Exclusive possession of land without title (personal property): Minister for the Army v Dalziel

Company shares: Bank of NSW v Commonwealth Confidential information protected by equitable remedies: Smith Kline v

Department of CS Choses in action (rights to sue): Georgiadis v Telecommunications Commission Statutory rights to government payments: Health Insurance Commission v

Peverill Real property (land) not merely regulating use and enjoyment of proprietary

rights: Tas Dam Taking control of a business by new board of directors: Banking Case Forced sale of goods: McClintock v Cth, TPC v Tooth, BMA v Cth.

o Acquisition requires transfer of a concrete benefit to the government or another person. Does not include regulation of property use. Deprived right not enough, someone else has to have gotten right/analogous.

31

Page 32: · Web viewoncerned ‘tax bonus payments’ made by Rudd government. Argued that payments were unsupported by a head of power. Upheld legislation as a valid exercise of s 61, together

UQ Semester 1, 2014o Property need not be transferred directly to the Commonwealth- could be third party:

Magennis v Commonwealth. However, must be an acquisition of property by someone under a Commonwealth law.

o Follows that compensation need not be paid for diminished property value resulting from:

Export restrictions: Murphyores v Commonwealth Land zoning: Trade Practices Commission v Tooth Price Controls: BMA v Commonwealth Restrictions in trademark use: BATA v Commonwealth

o Further illustration provided by Tasmanian Dam- right to build a dam. Mason, Murphy and Brennan JJ held there was no acquisition of property as there was no transfer of title. Deane J dissented- mere ability to restrict the use of the land could be considered an acquisition.

o Georgiadis v Telecommunications Commission (1994)- concerned legislation preventing workers from suing their employer (Cth) for injury under the common law. Conduct had already happened, law suit was being put in motion. Currently vested right, includes noncorporeal property. Majority held acquisition, as extinguishment of right conferred substantial benefit on Commonwealth.

o Health Insurance Commission v Peverill (1994)- concerned legislation retrospectively reducing Medicare benefits payable to doctors. Held not to be an acquisition, merely an adjustment of claims based on policy considerations. No-one acquired a corresponding benefit. Statutory scheme- going to be subject to adjustment from time to time. When they are adjusted, can’t say this is an acquisition of property.

o Difference not common law vs statute, but more to do with whether a right is subject to modification normally.

o ICM v Commonwealth (2009)- challenge to the replacement of bore licenses with aquifer access licenses. Change reduced permissible quantities of water. French CJ, Gummow, Hayne, Crenna, Kiefel and Bell JJ: no acquisition. Licenses susceptible to variation by statute and the government did not gain clear benefit. Heydon J dissented- licenses were property and had effectively been acquired.

o BATA v Commonwealth (2013)- Law regulated appearance of tobacco product packaging and the use of trademarks on packaging. Plaintiffs argued it extinguished IP rights. 6 judges held no acquisition of property, but merely regulation of its use. Heydon J dissented, noting legislation sterilized property rights and also compelled presence on packages of the Commonwealth’s messages.

o Cannot use another head of power to circumvent this right. Exception- transfer of property by law does not require compensation when the transfer is authorized by the very nature of the HOP.

o Example: law allowing use of IP of others. Expressly authorized by copyright power: Nintendo v Centronics.

o Tutorials: Will be an acquisition where: Real property (land) acquired by Commonwealth (Tasmanian Dam) There is a forced sale of goods to the Commonwealth (McClintock v

Commonwealth) Acquisitions are made by third parties and States as agents for the Cth under

Cth legislation (TPC v Tooth, Magennis v Commonwealth)

32

Page 33: · Web viewoncerned ‘tax bonus payments’ made by Rudd government. Argued that payments were unsupported by a head of power. Upheld legislation as a valid exercise of s 61, together

UQ Semester 1, 2014 A corporation is taken over by a new board of directors (Bank Nationalisation

Case)o No acquisition where:

Sale of goods is voluntary (TPC v Tooth, BATA Case) There is a restriction on the use and enjoyment of proprietary rights (Tas Dam,

BATA Case) Legislation merely alters/extinguishes statutory rights created by Cth law (ICM,

Cth v WMC) Role of s 96

o Can Cth make grant of financial assistant that requires the states to acquire property other than on just terms?

Magennis v Cth- struck down a statutory agreement for a state to acquire property not on just terms. However, Pye v Renshaw upheld an informal agreement for a similar purpose- not going to scrutinize the purpose too closely if it isn’t enshrined in legislation or something.

Majority in ICM and Spencer reiterated that a formal agreement under s 96 may breach s 51(xxxi) and left open the question of informal agreements. Indication of willingness to overturn Pye?

Just termso Do not necessarily require market value. However market value at time of acquisition

should be the starting point- Nelungaloo v Commonwealth.o Compensation required is understood as a fair amount that takes into account both

property owner’s interests and the public interest in the acquisition (Nelungaloo). Nothing less than full compensation will suffice- Georgiadis per Brennan J.

However exact amount leaves some room for discretion. Body determining compensation must be unbiased and give the property owner

a fair hearing. Must be reasonable provision for review of the decision- Apple and Pear Marketing Board v Tonking.

o HC has been critical of schemes where Cth leaves compensation to be determined by another party. Valid if:

Other party acted impartially and fairly (Andres v Howell) Owner had opportunity to make submissions or represent their interests on the

amount of compensation (Bank Nationalisation Case, Apple and Pear MB). Trial by jury (s 80)

o Whether to make an offence triable on indictment is up to Parliament: R v Archdall.o Strong criticism: eg Dixon and Evatt JJ: interpretation makes a mockery of s 80,

rendering its protection purely illusory: R v Lowenstein.o Nonetheless Archdall reaffirmed by 4 judge majority in Kingswell v R. Brennan & Deane

JJ dissented strongly.o s 80 can be circumvented by Cth, but cannot be waived by an accused person: Brown v

R.o Requires conviction by a unanimous jury verdict. Conviction by majority verdict

inconsistent with institution of trial by jury: Cheatle v R. Religious freedom (s 116). Two limbs

33

Page 34: · Web viewoncerned ‘tax bonus payments’ made by Rudd government. Argued that payments were unsupported by a head of power. Upheld legislation as a valid exercise of s 61, together

UQ Semester 1, 2014o (1) Establishment clause prohibits Commonwealth from establishing a national religion

or imposing religious observance. Also rules out imposing religious test for any office under Commonwealth.

o (2) Free exercise clause prevents the Commonwealth from interfering with free exercise of religious beliefs.

o Kruger v Commonwealth- s 116 does not apply to state laws.o Construed narrowly- broad understanding of religion, narrow interpretation of the

circumstances in which a law impacting religious practice will violate s 116.o Definition of religion- Jehova’s Witnesses Case per Latham CJ- belief in supernatural

reality & adherence to set of rules of conduct with some supernatural significance. Would Buddhism count?

o Establishment clause discussed in A-G (Vic); Ex rel Black v Commonwealth (DOGS Case)- concerned issue of whether it was constitutional for Commonwealth to fund religious schools. Upheld funding arrangement.

Barwick CJ and Wilson J - ‘establishment’ is confined to establishment of a religion as a national or Commonwealth institution.

Gibbs and Mason JJ - narrower still- ‘establishment’ only applies to designation of an official state religion.

Stephen J - broader- establishment clause prohibits discrimination between religions.

Murphy J- dissented- ‘establishment’ should be interpreted widely to ban any state assistance to religious institutions. Would have struck down funding arrangements.

o Free exercise clause considered in Krygger v Williams- objected to compulsory military service on religious grounds. HC was dismissive. Not well-reasoned. Seems to suggest that only laws that specifically target religious practices will offend s 116.

o HC returned in Jehova’s Witnesses Case. Latham CJ clarified that clause protects not only free exercise of religion, but also freedom not to have a religion at all.

o Freedom is not absolute- subject to limitations reasonably necessary for the protection of the community and in the interests of social order. Narrow interpretation. Point to some policy objective that justifies it.

o Revisited in Kruger- stolen generation government policy. Held policy did not violate s 116. Free exercise clause only prohibits laws intended to impact on the free exercise of religion. General laws that incidentally impact on religious freedom are permitted. Latham CJ had made similar suggestion in JWC.

An express right to vote? (s 41)o Looks like express albeit qualified constitutional right to vote.o King v Jones- held term ‘adult person’ did not cover <21. Case occurred when voting

age 21 federally, 18 SA.o R v Pearson; Ex Parte Sipka- held s 41 was merely transitional and no longer of any

effect. Majority took view section only protected voting rights of persons who were enfranchised under state law prior to adoption of a Cth franchise law in 1902. Note: does not contain “Until Parliament otherwise provides.”

34

Page 35: · Web viewoncerned ‘tax bonus payments’ made by Rudd government. Argued that payments were unsupported by a head of power. Upheld legislation as a valid exercise of s 61, together

UQ Semester 1, 2014

Week 11: Implied Constitutional Rights Constitutional implications examples- SOP, Rule of Law, Intergovernmental immunities, pre-

Engineers. Qualified right to freedom of political communication.

o The issue arose initially in two 1992 cases: Nationwide News v Wills and ACTV v Cth.o This right has been linked to the ‘directly chosen’ clause in ss 7 and 24. If citizens are

to have an effective choice, they must be free to discuss the performance of the candidates.

o Representative government implicit in the Australian constitutional order, also from ss 7/24. Freedom of political communication in relation to the discussion of public and political affairs/matters is indispensible to representative government and is, therefore implicit in the Constitution.

o NB: implied right of freedom of communication does not protect general right to freedom of speech

Nationwide News (1992):o The Australian published article attacking the IRC. Accused members of corruption.o Nationwide News was charged under s 299(1)(d)(ii) of the Industrial Relations Act 1988

(Cth), which made it an offence to publish information calculated to bring the Commission into disrepute.

o HC unanimously interpreted section as prohibiting criticism of IRC even if comments true/reasonable.

o Provision deemed unconstitutional. However, the reasoning of the judges varied. Majority (Brennan, Deane, Toohey and Gaudron JJ) relied on implied

constitutional right to freedom of communication. s 299(1)(d)(ii) was sufficiently connected to s 51 (xxxv) (arbitration). Powers are ‘subject to this Constitution’ implied freedom of political

communication. Emphasized that the implied freedom was subject to reasonable

restrictions. However, section imposed unreasonable limit, as it prohibited legitimate

criticism. Dawson J more conservative approach no implied right of political

communication. Based his decision on scope of arbitration power- characterization issues- struck down.

35

Page 36: · Web viewoncerned ‘tax bonus payments’ made by Rudd government. Argued that payments were unsupported by a head of power. Upheld legislation as a valid exercise of s 61, together

UQ Semester 1, 2014 The power in s 51 (xxxv) is purposive power. It covers conciliation and

arbitration for the prevention of industrial disputes. This type of power should be widely interpreted.

However, s 299(I)(d)(ii) was beyond the power conferred by s 51(xxxv), as it was disproportionate to the achievement of a legitimate end.

Mason CJ and McHugh J took a more moderate/middle view. Section 299(I)(d)(ii) lacked a sufficient connection with s 51(xxxv).

Freedom of speech in relation to public affairs and institutions is an issue to be taken into account in determining proportionality between an Act and the relevant head of power.

ACTV v Cth (1992):o Act prohibited electronic broadcasting of political advertisements at election times.o The prohibition applied to all federal and state elections and referenda.o Required free air time to be granted to all political parties based on representation in

previous parl.o Free time allocated in proportion to no. of 1st preference votes each party received at

last election.o Private citizens and independents could also obtain free time at the discretion of

tribunal.o The legislation was challenged on two grounds:

Violated the implied constitutional right to freedom of communication; Unconstitutional burdening of the states that impaired their capacity to function.

o Majority (Mason CJ, Deane, Toohey, Gaudron, Brennan and McHugh JJ; Dawson J dissenting) accepted that the Constitution contained an implied guarantee of freedom of political communication.

Five of majority (Brennan J dissenting) ruled unreasonable restriction on that freedom.

Brennan and McHugh JJ held that Part IIIB unconstitutionally burdened the States.

Basis of the Implied Right:o The majority judges in ACTV and Nationwide News followed a two stage process of

reasoning: The constitutional provisions reveal an intention to set up a representative form

of government. Freedom of communication is essential to the functioning of this form of

government. In a unanimous joint judgment in Lange v ABC (1997), the HC clarified that the freedom

derived from the words ‘directly chosen by the people’ in ss 7 and 24. End of Dawson J’s resistance to the implied right.

o ss 7 and 24 do not confer personal rights on individuals. Preclude curtailment of freedom.

Political communication:o Implied right only covers communication on political matters and not other types of

speech. Political communication includes discussion of government policy or debate on

the performance of public officials.

36

Page 37: · Web viewoncerned ‘tax bonus payments’ made by Rudd government. Argued that payments were unsupported by a head of power. Upheld legislation as a valid exercise of s 61, together

UQ Semester 1, 2014o The limits of the right are illustrated by Cunliffe v Cth (1994). Concerned a provision in

the Migration Act stating that only a registered migration agent could provide advice on migration matters. Majority held that this was not a restriction on political communication, as advice on migration issues was not a political matter.

Actions as Communication:o Can actions alone amount to constitutionally protected communication? Levy v Victoria

(1997): Levy protested against duck shooting by entering a hunting area. Charged and

convicted under a Victorian law which prohibited people from entering designated hunting areas.

Levy argued he was exercising his freedom of political communication. The HC accepted that actions may amount to communication. The law restricted Levy’s political communication. However, it was held to be a reasonable restriction, since it protected public safety and was a proportionate measure to take in fulfillment of that objective.

Subject to reasonable restrictions. One relevant factor- whether it goes further than is necessary to pursue policy objective, and whether it leaves any other avenues for people wishing to exercise the freedom.

Political Speech in the States:o HC has held that the freedom of political communication applies to state laws as well.

ACTV, Mason CJ, Deane, Toohey and Gaudron JJ noted that political communication can’t be subdivided to correspond with tiers of government. National political discourse covering both.

Similarly, the majority in Stephens v Western Australian Newspapers held that the freedom extends to discussion of the conduct of members of a state legislature.

Another example of the freedom being applied to state laws is Coleman v Power Legitimate Restrictions:

o Freedom is not absolute. Subject to restrictions that are reasonably adapted to a legitimate end.

o It is useful to distinguish here between restrictions on content and restrictions on mode.

o Mason CJ and McHugh J in ACTV: restrictions on content more difficult to justify than mode.

This is because restrictions on content go directly to the ideas being communicated. ACTV was mode, Nationwide News was content. Both nevertheless struck down.

o Test for working out if a restriction on political speech is permissible (Lange). (1) Does the law impose a burden on political speech? (2) Is the restriction a reasonable one?

(a) Is it aimed at a legitimate government objective? (b) Is it reasonably appropriate and adapted to the objective?

o This means the restriction must not be excessive. The judges will look at whether there were any less extreme means available for achieving the same objective.

o In deciding whether a restriction is appropriate, the court will take account of the need to protect the institutions of representative and responsible government.

37

Page 38: · Web viewoncerned ‘tax bonus payments’ made by Rudd government. Argued that payments were unsupported by a head of power. Upheld legislation as a valid exercise of s 61, together

UQ Semester 1, 2014 Coleman v Power (2004)

o Issue of reasonable restrictions arose. Coleman distributed leaflets insulting police officers.

The leaflets contained anti-police comments. When Constable Power asked for one of the leaflets Coleman refused,

proclaiming loudly, ‘This is Constable Brendan Power, a corrupt police officer’. Coleman was arrested and charged under s 7(1)(d) of the Queensland Vagrants

Act for using ‘threatening, insulting or abusive words’ in a public place. Section 7(1)(d) read down by the HC and Coleman’s conviction was reversed.

The parties agreed that s 7(1)(d) restricted free political speech. McHugh, Gummow, Hayne and Kirby JJ recognised that insults may be political communication, if they are intended to make a political point.

McHugh J held that a prohibition on insults, as opposed to threats or abuse, was too broad to be considered reasonably adapted to a legitimate end. He read down the section to remove the reference to insults.

Gummow, Hayne and Kirby JJ went further. They held that the provision was valid only in its application to words that were likely to provoke violence. Insofar as it extended to other types of language, it was unconstitutional.

Gleeson CJ, Callinan and Heydon JJ dissented. Wotton v Qld [2012]

o Further illustrates the issue of reasonable restrictions. Wotton participated in a protest on Palm Island following an indigenous death in custody. He was charged and sentenced to 6y with 2y non-parole.

o Later granted parole on the condition that he not attend public meetings on Palm Island without approval and that he not communicate with the media.

o French CJ, Gummow, Hayne, Crennan, Bell and Kiefel JJ held that the conditions burdened political speech, but were appropriate and adapted to public safety.

o Heydon J held that the conditions did not ‘realistically threaten’ freedom of speech. Monis v The Queen [2013]

o The defendants sent abusive letters to the families of Australian soldiers killed in Afghanistan.

o Charged under federal law with using a postal service ‘in a way ... that reasonable persons would regard as being, in all the circumstances ... offensive’.

o HC split 3-3 on whether the offence was consistent with the implied freedom. Lower decision affirmed.

o Crennan, Kiefel and Bell JJ read the offence as only applying to ‘seriously offensive’ communications. This was held to be consistent with the implied freedom upheld conviction.

o French CJ, Hayne and Heydon JJ struck down the law and overturned conviction (although Heydon J did so only reluctantly – undesirable result – forced by ridiculousness of freedom).

Brown v Members of Classification Review Board (1998)o FC rejected argument that an article in a student magazine, instructing readers how to

shoplift was protected by the implied freedom because it had been motivated by redistributive economic theory.

38

Page 39: · Web viewoncerned ‘tax bonus payments’ made by Rudd government. Argued that payments were unsupported by a head of power. Upheld legislation as a valid exercise of s 61, together

UQ Semester 1, 2014o Held that even if accepted that article had political motivation, the prohibition

publication of material designed to facilitate crime was reasonable limitation on the freedom of political communication.

The Defence of Qualified Privilege: o Two 1994 cases, majority of the HC (Mason CJ, Deane, Toohey and Gaudron JJ) created

an expanded defence of qualified privilege, relying on the constitutional freedom of communication.

Theophanous concerned a critical newspaper letter about Theophanous, an MP. Stephens concerned criticism of a parliamentary study trip published in the West

Australian.o The defence was held to apply where:

Imputation concerns political matter and refers to a public officer/candidate for public office;

There was no awareness of falsity and the statement was not reckless or unreasonable.

HC clarified the defence in its unanimous joint judgment in Lange v ABC.o Held that the defence of qualified privilege has four elements:

The communication must be disseminated to a wide audience; It must relate to a political matter; It must be reasonable; and It must not be malicious.

o A communication will be reasonable if: There are reasonable grounds to believe it is true; The publisher took reasonable steps to verify the facts; The publisher did not believe it to be untrue; and Publisher sought response from the person targeted, unless

impractical/unnecessary to do so.o Malice will defeat the defence. Requires proof communication motivated by improper

purpose. Intention to cause political damage is not an improper purpose.o Common law and statutes may expand the defence, but they may not lawfully restrict

it. Freedom of Association:

o Several HC judges hinted at the possibility of an implied right to freedom of association. ACTV:

Gaudron J : Parl. democracy may entail freedom of movement/association/speech generally

McHugh J : Logical conclusion to be drawn from ss 7/24 is that freedom of participation, association and communication in relation to federal elections is protected by the Constitution.

o Yet to be endorsed by a majority of the HC. Would be subject to reasonable restrictions.

The Right to Vote:o Some recent cases comes close to endorsing implied right to vote (subject to

reasonable restrictions).o Roach v Electoral Commissioner (2007)

39

Page 40: · Web viewoncerned ‘tax bonus payments’ made by Rudd government. Argued that payments were unsupported by a head of power. Upheld legislation as a valid exercise of s 61, together

UQ Semester 1, 2014 Struck down blanket ban on prisoners voting in Cth elections even if only short

sentences. Gleeson CJ : ‘the words of ss 7 and 24, because of changed historical

circumstances including legislative history, have come to be a constitutional protection of the right to vote’.

Interesting because he describes it as a ‘right’ and also because he seems to be suggesting that this right has emerged over time.

Others reached same outcome with cautious language. Indicated support for such a position.

Also held that although ss 8 and 30 of the Constitution confer on Parliament the power to prescribe qualifications for voting, the fact that the Constitution is founded upon a system of representative democracy means that this power is not unconstrained only limitations on voting that have some rational basis and survive the proportionality test would be constitutionally valid.

o See also Rowe.

Week 12: State Constitutional Change Legislative power- power to make ordinary laws under the Constitution. Constituent power- power to make a law that changes [or creates] the Constitution. Parliaments of Australian States have plenary legislative power to ‘make laws for the peace,

welfare and good government’- see for example s 2 Constitution Act 1867 (Qld). Limits on State legislative power (in addition to s 109)

o Colonial Laws Validity Act 1865 (UK)- Colonial law must not be repugnant to Imperial Law (s 2). UK Acts apply to colonies if extended by express words or necessary implication (s 1).

o Australia Acts 1986 confirmed UK Parliament no longer has legislative power over States (s 1).

40

Page 41: · Web viewoncerned ‘tax bonus payments’ made by Rudd government. Argued that payments were unsupported by a head of power. Upheld legislation as a valid exercise of s 61, together

UQ Semester 1, 2014o s 2(1) confirmed plenary power. s 3: State law will no longer be void for inconsistency

with UK statute. ss 8 and 9: a State law shall not be disallowed by monarch or reserved for approval.

State constituent powero CLVA s 5- Plenary power, subject to laws being passed in such manner & form as

required.o Factors- a) Constitution is merely an Act, b) they have plenary power, c) s 5 of the

CLVA.o CLVA provided that colonial (and later State) Parliaments have power to change

Constitutions by an ordinary statute.o Confirmed in McCawley’s Case- State Parliaments may make laws that are inconsistent

with their Constitutions without passing a formal amendment.o However s 5 CLVA- when law concerns “constitution, powers or procedure” of

Parliament must follow manner and form requirements. Manner and Form limitations

o Replaced by s 6 of AA, which two key changes: “made in such manner and form” [allows referendum requirements] & “required by an Act of that Parliament”.

o Allows State Parliaments to prescribe special procedures for future constitutional changes.

o Normally, a statute imposing special process would not bind future Parliaments. General rule- plenary power- can repeal/override any previous law.

o s 6 AA takes precedence over ordinary State Laws- generally accepted that it gives paramount force to State manner and form requirements that fall within scope. But why? See later.

“Constitution, powers and procedure”o Manner & form requirements can only be imposed on laws relating to this if they are to

be enforced.o Changes to powers of executive branch do not concern this- Trethowan’s Case (Dixon

J). Judiciary too (presumably).o Change to qualifications of members of Parliament does not concern ‘constitution’ of

legislature- Clydesdale v Hughes. Reasoning is cursory; not much discussion. Seems to rest on narrow interpretation of ‘constitution’.

o However, view was affirmed (obiter) by Wilson J in WA v Wilsmore.o A-G (WA) v Marquet- bill changed electoral districts and regions. Held: changed the

constitution of the Parliament. Comments regarding ‘constitution’- s 6 not to be read as confined to laws which abolish/take away representative character of a House/Parliament. ‘Constitution’ extends to features which give it a representative character- for example different methods of election. Each would be representative but Parliament would be differently constituted.

o ‘Powers’ include legislative power, power to punish for contempt, power to conduct public inquiries etc

o ‘Procedure’ refers to the procedural rules governing Parliament and its bodies. Procedure must also be a valid manner and form requirement.

o M & F requirement not valid if it requires future changes to be approved by a body external to the legislature- Comalco Case; West Lakes Case.

41

Page 42: · Web viewoncerned ‘tax bonus payments’ made by Rudd government. Argued that payments were unsupported by a head of power. Upheld legislation as a valid exercise of s 61, together

UQ Semester 1, 2014o Comalco- was a requirement that future changes to legislation on mining leases be

approved by Comalco board. Didn’t fall under s 6/s 5 CLVA because not constitutional matter, but was held that because requirement not legislative in nature, could not come under s 6/s 5 CLVA.

o West Lakes- King CJ- requiring consent of something other than Parliament or people is not a manner and form requirement, but a renunciation pro tanto of law-making power. Relates to substance of power, and not the manner and form of its exercise.

o However, a referendum is a valid M & F requirement- Trethowan’s Case. Rich J- s 5 CLVA relates to entire process of turning a proposed law into an Act. In West Lakes, King CJ- requirement for referendum, although extra-Parliamentary, is easily seen to be a M & F provision because it is confined to obtaining the direct approval of the people whom the legislature represents.

o s 6 AA removed any doubt- replaced “passed” with “made”.o Exception to the rule- requires consent of external body- people.o Not valid if it is so onerous it amounts to an abdication of power- West Lakes Case. King

CJ: Not valid if properly viewed as an attempt to deprive Parliament of powers rather than a measure to prescribe the manner and form of their exercise.

o King CJ : M & F requirements relating to fundamental constitutional provision may legitimately be more onerous than other requirements relating to less fundamental matters.

Mandatory or directory?o May be mandatory in some cases- need only be a genuine and reasonable attempt.o M & F process that can be unilaterally derailed by minority interests may be construed

as merely directory: Clayton v Heffron. Concerned a process of resolving deadlocks between NSW Parliament Houses. Process required conference of all party managers.

o HC held process merely directory, since if it was mandatory it could be aborted unilaterally by any one party.

Double entrenchmento Will a M & F requirement prevent future changes to requirement itself?o Arguable that section providing the M & F requirement could be enforced anyway,

given s 6.o However double entrenchment makes requirement more secure. Referendum M & F

requirement not being enforced on basis of referendum section, but by s 6 AA. Manner and Form in Queensland

o Qld Constitution unique provision- s 53- provides that any changes to the office of Governor or ss 1, 2, 2A, 11A, 11B, or 53 of the Act require a referendum. These concern composition and powers of legislature and the executive.

o Questionable whether valid in application to the executive- Dixon J in Trethowan.o s 53’s application to s 2 means Qld Parliament cannot impose any additional M & F

requirements without referendum, since this would change powers of legislature.o S 53(5) gives all Qld voters standing to enforce M & F provisions in SC. Normally only

those individuals directly and personally affected (above general public) have standing to challenge on constitutional grounds. s 53(5) empowers SC to grant injunctions to prevent M & F being ignored. May not be possible in other States without similar provisions- Clayton v Heffron.

Manner and Form beyond s 6 AA42

Page 43: · Web viewoncerned ‘tax bonus payments’ made by Rudd government. Argued that payments were unsupported by a head of power. Upheld legislation as a valid exercise of s 61, together

UQ Semester 1, 2014o Actual example might be s 53 insofar as it applies to Governor (see Trethowan’s Case).o Question about whether sovereign Parliament can bind itself.

View 1- Sovereign can do anything within territorial jurisdiction, including limiting its own power. Could remove some of its own powers and create a new legislative body. Could vote itself out of existence.

View 2- Sovereign can freely repeal past acts, ignoring limits it has placed on itself.

o Ultimately depends on willingness of Courts to enforce restrictions. Have been some judicial comments suggesting that a Parliament could reconstitute itself for particular purposes, thereby limiting its future powers:

Trethowan- Rich J- general legislative power of NSW Parliament was sufficient to reconstitute itself for particular purposes.

Trethowan- Dixon J- even sovereign UK Parliament could bind itself by manner and form requirements if courts had power to intercede when they are not met.

Fox Hunting Case- Lord Steyn- Parliament acting as ordinarily constituted may functionally redistribute legislative power in different ways. May for specific purposes provide for a 2/3 majority in houses. This is a redefinition of Parliament for a specific purpose.

Fox Hunting Case- Baroness Hale- No reason why Parliament should not decide to re-design itself, either in general or for a particular purpose.

o Australian State Legislatures creatures of positive enactments that courts, with their full judicial review powers, have a duty to interpret and enforce. May enforce non-s 6 AA M & F requirements.

o General academic consensus for other view. Can’t limit power of future Parliament unless it harnesses the power of s 6 AA or some other source.

o Also raises public policy issues- could lead to destruction of parliamentary democracy- each party would seek to entrench its own policies. On the other hand, might be good to entrench some fundamental features of the Constitution such as powers and independence of judiciary.

Why is s 6 binding?o AA passed by Cth and UK Parliaments to avoid doubt about Cth’s capacity to legislate

unilaterally. Constitutional basis for Cth Act claimed under s 51(xxxviii)- all States consented through Acts.

o Can be argued UK AA invalid because Imperial Parliament did not have power to change Australia’s constitution. But it did under request and consent procedure under SOW?

o Kirby J took view in A-G (WA) v Marquet that AA (Cth) invalid for inconsistency with s 106 (giving power over State Constitutions to States).

o Where does force of AA’s come from? UK Parliament? Cth (under s 51)? States? Combined?

o State Parliaments can’t bind themselves either.o What does each of these possibilities mean for the ultimate capacity of the AA’s to bind

State Parliaments?

43

Page 44: · Web viewoncerned ‘tax bonus payments’ made by Rudd government. Argued that payments were unsupported by a head of power. Upheld legislation as a valid exercise of s 61, together

UQ Semester 1, 2014

Week 13: Commonwealth Constitutional Change Constitutions range from rigid (eg German Basic Law) to flexible (eg UK). Aus States- flexible

end. s 128- two-step process for amending Constitution.

o Proposed change must be approved by absolute majority of both Houses. May be bypassed if one House twice rejects/fails to pass an amendment by the other House.

o Proposed amendment must go to popular vote at referendum. Must be passed by a) majority of electors overall (including Territories) and b) a majority of electors in a majority of States. Implements both federalism and representative democracy.

Judicial amendmento Constitutional provisions often vague. Courts will have to clarify scope in applying them

to cases.o Interpretation has sometimes brought about important constitutional changes- eg

ACTV.o Proper role of judiciary in shaping Constitution is controversial.

Constituent powero Legislative body is Parliament, constituent body is Parliament + electorate: s 128.

Concept of sovereigntyo Which body has ultimate power within a particular legal system?o Australian Constitution created by UK Act. Could UK Parliament amend it?o On a traditional view of sovereignty:

All persons are subject to sovereign’s authority44

Page 45: · Web viewoncerned ‘tax bonus payments’ made by Rudd government. Argued that payments were unsupported by a head of power. Upheld legislation as a valid exercise of s 61, together

UQ Semester 1, 2014 The sovereign is not bound by another authority The sovereign is not bound by its own authority

o UK Parliament traditionally viewed as sovereign. Its formal power was unlimited (including by unwritten constitution). Ongoing debate over impact of EU Law on UK sovereignty. Also Scotland issue

Limits of sovereigntyo Territorial limits.o Practical limits. Consider Eldest Child Act.o Moral limits? Can sovereign pass law that is repugnant to basic moral values? o Sometimes a sovereign body is practically unable to undo what it has previously done.

For example, once UK Parliament grants (de jure or de facto) independence to a country, it may not obey Parl.

Hans Kelsen and the Grundnormo Norm is an ‘ought’ proposition. Legal norm has validity conferred upon it by higher

legal norm.o All form part of unified system. Trace validity up legal hierarchy Grundnorm.o Represents ultimate source of constitutional authority.o Grundnorm: You ought to be obey the constitution\any norm validly incorporated into

it.o Transcendental-logical presupposition. Idea that lies behind entire system of legal

rules.o Just above constitution; assumption that constitution legally binding.o Grundnorm may change through revolution or evolve through constitutional change.

Change occurs when legal officials and citizens accept a new conception of the ultimate source of sovereignty.

Adventures of the Australian Grundnormo 1788-1823: Governor virtual dictator. But must obey royal orders and Imperial

legislation. Legislative power: Governor, Monarch and Imperial Parliament Constituent power: Monarch and Imperial Parliament Grundnorm- ought to obey Monarch and Imperial Parliament

o 1823-1828: NSW Legislative Council established (other colonies follow suit). Governor should act on its advice. UK retains ultimate legislative power.

Legislative power: Governor, Legislative Council and Imperial Parliament Constituent power: Imperial Parliament

o 1828-1865: Australian Courts Act 1828 (UK) deprives Governor of any residual power to override Legislative Council. English law made applicable in colonies as it existed on 25 July 1828. After that date, imperial laws apply only by paramount force (if extended to the colony expressly or by necessary implication). Colonies progressively receive Constitution Acts from UK Parliament.

Legislative power: Colonial and Imperial Parliaments Constituent power: Imperial Parliament

o 1865-1900: CLVA 1865 (UK) clarifies application of Imperial legislation to the colonies. UK Acts apply to colonies if extended by express words or necessary implication:

s 1 Colonial laws that are repugnant to Imperial Acts are void and inoperative: s 2

45

Page 46: · Web viewoncerned ‘tax bonus payments’ made by Rudd government. Argued that payments were unsupported by a head of power. Upheld legislation as a valid exercise of s 61, together

UQ Semester 1, 2014 Colonial legislatures have power to make laws with respect to a) courts b) the

constitution, powers and procedure of the legislature: s 5 Legislative power: Colonial and Imperial Parliaments Constituent power: Colonial Parliaments and (ultimately) Imperial Parliament.

o 1900-1931: Colonies agree to federate under CACA (UK). Constitution set out in s 9. Imperial Parliament retains power to make laws for Commonwealth and States.

Legislative power: State and Commonwealth Parliaments and Imperial Parliament (insofar as CLVA is in force)

Constituent power: Section 128 and (ultimately) Imperial Parliament- enacted CACA.

o 1931-1986: SOW 1931 (UK) (adopted in 1942) further limits role of Imperial Parliament. CLVA no longer applies to Dominions (inc Australia): s 2. Request and consent procedure- s 4.

Legislative power- State and Commonwealth Parliaments, and Imperial Parliament, following request and consent.

Constituent power- s 128; Imperial Parliament following request and consent. Repeal of the Statute of Westminster?

o UK Parliament voluntarily limited its own power to legislate with respect to Australia. But said to be sovereign. Could it simply repeal s 4 and legislate for Australia without request?

o Answer depends on whether Australian Courts/other authorities would recognize repeal as valid and binding. Really a question about the Grundnorm: Does the Australian Grundnorm still recognize the UK Parliament as the ultimate constituent authority? Or has it irrevocably shifted to domestic sovereignty?

Australia Acts 1986o Object: sever remaining constitutional links (except monarchy). In accordance with

SOW, identical Acts passed by both UK and 7 Australian Parliaments.o No UK Acts to apply in Australia: s 1. s 12 repealed request and consent procedure.o States have full legislative power to override imperial legislation: ss 2 and 3.o State Governors have full executive powers as representatives of Queen. Queen may

not overrule state laws: ss 7 and 8.o State Governors must not be required to withhold assent to state laws: s 9.o UK government no longer has any responsibility for the government of the States: s 10.o Appeals to Privy Council from Australian Courts are abolished: s 11.o AA itself only to be repealed or amended in accordance with s 15. Must be done by

unanimous consent of all 7 Parliaments. o Nothing in s 15 prevents exercise by Cth Parliament of power power conferred by s

128: s 15(3) Grundnorm after 1986

o UK Parliament seems to voluntarily relinquish its last vestiges of potential authority over Aus.

o Could UK repeal its AA and then change Australian law?o Seems likely that ultimate constituent authority in Australia, at least since 1986

(arguably long before) is no longer UK Parliament. Could no longer assert authority. Legislative power- State and Commonwealth Parliaments. Constituent power- Commonwealth Parliament and electorate: s 128.

46

Page 47: · Web viewoncerned ‘tax bonus payments’ made by Rudd government. Argued that payments were unsupported by a head of power. Upheld legislation as a valid exercise of s 61, together

UQ Semester 1, 2014 An alternative to s 128?

o Christopher Gilbert- argued that s 15 AA (UK) creates alternate way of amending Constitution.

o Two steps: 1. Commonwealth and State Parliaments amend s 15 of AA (UK) to allow

amendment of CC. 2. Australian Parliaments may then use s 15 to amend Constitution.

o Does this work? Is it consistent with Australian Grundnorm? Note: proposal effectively involves Australian Parliaments harnessing the power

of the UK Parliament (by way of the AA) and then using it to amend the Australian Constitution.

Could the UK Parliament itself amend the Australian Constitution? If not, how can the Australian Parliaments use the AA (UK) to do so?

Could the AA’s themselves be viewed as creating a new source of constituent power?

Can Australia become a republic?o Some commentators argued that not possible for Australia to become a republic

without involving the UK Parliament. s 128 does not expressly confer power to amend CACA.

o CACA refers to monarch. Not possible to change under s 128; UK Parliament would need to do so.

o Suppose Constitution changed under s 128 to remove all references to Queen. Would anybody notice/care what the covering clauses said?

o Suppose s 128 used to amend CACA. Would effectiveness be open to serious challenge?

o Would seem to be a question about Australian Grundnorm. What is the ultimate source of constituent authority Australia: UK Parliament or Australian people?

o Arguably: s 128 Parliament + people power to change anything about our constitutional system could change covering clauses.

47